Wikipedia:Reference desk/Science: Difference between revisions

From Wikipedia, the free encyclopedia
Content deleted Content added
removing medical question and suggested diagnosis; see Wikipedia_talk:Reference_desk
Line 97: Line 97:
::::: Were it not for security worries, it would have made sense to use the far from ancient and doddering mind of Einstein in the defeat of the Axis, if he were willing to help. Einstein was only 63 in 1942. Richard Feynman was 68 when he played an important role in the Challenger shuttle disaster investigation, which was in an area of technology he had zero background in. Even if Einstein was squeamish about building bombs to drop on Hitler, there were many areas of war research he could have helped with, just as Thomas Edison worked on [http://www.worldwar1.com/sfedsub.htm acoustic detection antisubmarine apparatus and synthetic phenol] in WW1, 1914-1918, from age 67 to 71. At a mundane level, Einstein could have screened patent applications to pick out those which actually were physically practical and which could have use in war equipment or in any area useful to the war effort such as communication, electricity, transportation, food preservation, or refrigeration. [[User:Edison|Edison]] ([[User talk:Edison|talk]]) 16:30, 4 March 2011 (UTC)
::::: Were it not for security worries, it would have made sense to use the far from ancient and doddering mind of Einstein in the defeat of the Axis, if he were willing to help. Einstein was only 63 in 1942. Richard Feynman was 68 when he played an important role in the Challenger shuttle disaster investigation, which was in an area of technology he had zero background in. Even if Einstein was squeamish about building bombs to drop on Hitler, there were many areas of war research he could have helped with, just as Thomas Edison worked on [http://www.worldwar1.com/sfedsub.htm acoustic detection antisubmarine apparatus and synthetic phenol] in WW1, 1914-1918, from age 67 to 71. At a mundane level, Einstein could have screened patent applications to pick out those which actually were physically practical and which could have use in war equipment or in any area useful to the war effort such as communication, electricity, transportation, food preservation, or refrigeration. [[User:Edison|Edison]] ([[User talk:Edison|talk]]) 16:30, 4 March 2011 (UTC)


::::::Well, to me the real question is to ask whether he could have worked on ''other'' projects, and why he wasn't used more readily there. There were plenty of projects of far lesser security concern than the bomb — things that were graded only "secret" or "confidential" rather than "top secret." An indeed, Einstein did do ''some'' consulting with the Navy. But I just don't seem him packing up and going to a government lab, myself. (And, entirely separately, I would argue that Feynman's background — including his war work! — actually made him much more qualified for a Challenger-like investigation than Einstein would have been. Feynman had lots and lots of practical experience in debugging large systems; that's actually what he did during his work with Oak Ridge during the war on radiation safety, on the possibility of critical masses forming inadvertently, if I recall. I don't see any similar experience on Einstein's resume. Einstein and Feynman were both great theoretical physicists, but they had very different minds. Feynman's strikes me as the far more practically oriented.)
::::::Well, to me the real question is to ask whether he could have worked on ''other'' projects, and why he wasn't used more readily there. There were plenty of projects of far lesser security concern than the bomb — things that were graded only "secret" or "confidential" rather than "top secret." An indeed, Einstein did do ''some'' consulting with the Navy. But I just don't seem him packing up and going to a government lab, myself. (And, entirely separately, I would argue that Feynman's background — including his war work! — actually made him much more qualified for a Challenger-like investigation than Einstein would have been. Feynman had lots and lots of practical experience in debugging large systems; that's actually what he did during his work with Oak Ridge during the war on radiation safety, on the possibility of critical masses forming inadvertently, if I recall. I don't see any similar experience on Einstein's resume. Einstein and Feynman were both great theoretical physicists, but they had very different minds. Feynman's strikes me as the far more practically oriented.)
::::::Incidentally, there was, I do recall, a physicist who screened said patent applications for the Office of Scientific Research and Development in the manner that you describe ([[William A. Shurcliff|but we lack an article on him]]). It probably would have been a job Einstein could have, in fact, done quite well if he had been asked to, and wanted to. --[[User:Mr.98|Mr.98]] ([[User talk:Mr.98|talk]]) 16:51, 4 March 2011 (UTC)
::::::Incidentally, there was, I do recall, a physicist who screened said patent applications for the Office of Scientific Research and Development in the manner that you describe ([[William A. Shurcliff|but we lack an article on him]]). It probably would have been a job Einstein could have, in fact, done quite well if he had been asked to, and wanted to. --[[User:Mr.98|Mr.98]] ([[User talk:Mr.98|talk]]) 16:51, 4 March 2011 (UTC)
::::::::Physicists at MIT dropped everything to work on radar(secretly). By contrast, it would have been a morale builder if some aging science superstars such as Einstein were shown in Life magazine in a lab somewhere "doing war research" even if it was not really the forefront. A mathematician or physicist is typically far over the hill in terms of major breakthroughs by his early 60's but they are the bedrock of review panels, with experience and perspective to decide what is promising, and what is a blind alley that has been gone down before. I knew women who drove over an hour each way to work a long shift building artillery shells and airplanes in WW2, and it seems unconscionable to have brilliant physicists "sit the war out" rather than making whatever contribution they were capable of to keep Hitler and Hirohito from dividing up the spoils after defeating the Allies. [[User:Edison|Edison]] ([[User talk:Edison|talk]]) 16:59, 4 March 2011 (UTC)
::::::::Physicists at MIT dropped everything to work on radar(secretly). By contrast, it would have been a morale builder if some aging science superstars such as Einstein were shown in Life magazine in a lab somewhere "doing war research" even if it was not really the forefront. A mathematician or physicist is typically far over the hill in terms of major breakthroughs by his early 60's but they are the bedrock of review panels, with experience and perspective to decide what is promising, and what is a blind alley that has been gone down before. I knew women who drove over an hour each way to work a long shift building artillery shells and airplanes in WW2, and it seems unconscionable to have brilliant physicists "sit the war out" rather than making whatever contribution they were capable of to keep Hitler and Hirohito from dividing up the spoils after defeating the Allies. [[User:Edison|Edison]] ([[User talk:Edison|talk]]) 16:59, 4 March 2011 (UTC)
Line 106: Line 106:
::::::::::The problem is that there is literally zero reason to suspect this, other than "it'd be interesting if it were true." --[[User:Mr.98|Mr.98]] ([[User talk:Mr.98|talk]]) 20:27, 4 March 2011 (UTC)
::::::::::The problem is that there is literally zero reason to suspect this, other than "it'd be interesting if it were true." --[[User:Mr.98|Mr.98]] ([[User talk:Mr.98|talk]]) 20:27, 4 March 2011 (UTC)
:::::::::::Oh, you're absolutely right. But there are many things from that war that remain absolutely secret, and thus many histories that seem a little implausible. Like [[Glenn T. Seaborg]]'s discovery of so many elements ''after'' the war, as opposed to, hypothetically, while trying to build a better [[neutron initiator]]. We have to indulge our imaginations now and then in order to avoid ''believing'' systematic error is fact. [[User:Wnt|Wnt]] ([[User talk:Wnt|talk]]) 00:23, 5 March 2011 (UTC)
:::::::::::Oh, you're absolutely right. But there are many things from that war that remain absolutely secret, and thus many histories that seem a little implausible. Like [[Glenn T. Seaborg]]'s discovery of so many elements ''after'' the war, as opposed to, hypothetically, while trying to build a better [[neutron initiator]]. We have to indulge our imaginations now and then in order to avoid ''believing'' systematic error is fact. [[User:Wnt|Wnt]] ([[User talk:Wnt|talk]]) 00:23, 5 March 2011 (UTC)
::::::::::::I'm not sure I gather your Seaborg analogy at all — Seaborg discovered all those elements because he had a total monopoly on the tools necessary to discover said elements. There's no real mystery there. It's also not like he wasn't working on the weapons program at the same time — he was, in various capacities. While it's always possible there are secret programs out there that have not yet been discovered, there is really no reason to assume any science fiction. It's also hard to imagine that programs of the scale of what you're describing would have existed with absolutely ''no'' evidence having surfaced thus far. --[[User:Mr.98|Mr.98]] ([[User talk:Mr.98|talk]]) 02:24, 5 March 2011 (UTC)
::::::::::::I'm not sure I gather your Seaborg analogy at all — Seaborg discovered all those elements because he had a total monopoly on the tools necessary to discover said elements. There's no real mystery there. It's also not like he wasn't working on the weapons program at the same time — he was, in various capacities. While it's always possible there are secret programs out there that have not yet been discovered, there is really no reason to assume any science fiction. It's also hard to imagine that programs of the scale of what you're describing would have existed with absolutely ''no'' evidence having surfaced thus far. --[[User:Mr.98|Mr.98]] ([[User talk:Mr.98|talk]]) 02:24, 5 March 2011 (UTC)


== Time-travel photography ==
== Time-travel photography ==
Line 426: Line 426:
::Microscope, not telescope. Unlikely, for different reasons. [[User:Plasmic Physics|Plasmic Physics]] ([[User talk:Plasmic Physics|talk]]) 08:36, 6 March 2011 (UTC)
::Microscope, not telescope. Unlikely, for different reasons. [[User:Plasmic Physics|Plasmic Physics]] ([[User talk:Plasmic Physics|talk]]) 08:36, 6 March 2011 (UTC)


:It's also worth noting that you can't really tell the ''precision'' of those images very well from the image alone. At the level of an atom, uncertainty principle says there will be limits to your precision — not that you can't know anything. It's a ratio of how precise your understanding of both position and momentum can be. It is only when you get to the level of, say, a bare electron, that the ratio gets so extreme that you start being able to know only one or the other. It is a common entry-level physics problem to ask students to calculate how much precision in position/momentum you can have for visualizing things on that level, how "in focus" they can maximally be. --[[User:Mr.98|Mr.98]] ([[User talk:Mr.98|talk]]) 14:36, 6 March 2011 (UTC)
:It's also worth noting that you can't really tell the ''precision'' of those images very well from the image alone. At the level of an atom, uncertainty principle says there will be limits to your precision — not that you can't know anything. It's a ratio of how precise your understanding of both position and momentum can be. It is only when you get to the level of, say, a bare electron, that the ratio gets so extreme that you start being able to know only one or the other. It is a common entry-level physics problem to ask students to calculate how much precision in position/momentum you can have for visualizing things on that level, how "in focus" they can maximally be. --[[User:Mr.98|Mr.98]] ([[User talk:Mr.98|talk]]) 14:36, 6 March 2011 (UTC)


= March 6 =
= March 6 =
Line 505: Line 505:
When a woman gets bloated during her period, is the bloating all blood or water? <span style="font-size: smaller;" class="autosigned">—Preceding [[Wikipedia:Signatures|unsigned]] comment added by [[Special:Contributions/76.169.33.234|76.169.33.234]] ([[User talk:76.169.33.234|talk]]) 05:57, 6 March 2011 (UTC)</span><!-- Template:UnsignedIP --> <!--Autosigned by SineBot-->
When a woman gets bloated during her period, is the bloating all blood or water? <span style="font-size: smaller;" class="autosigned">—Preceding [[Wikipedia:Signatures|unsigned]] comment added by [[Special:Contributions/76.169.33.234|76.169.33.234]] ([[User talk:76.169.33.234|talk]]) 05:57, 6 March 2011 (UTC)</span><!-- Template:UnsignedIP --> <!--Autosigned by SineBot-->
:It's [[water retention]]. Surprisingly little blood is lost during the menstrual period - about a teacup over the average 5 days. Most of what you see is [[endometrium]], or the lining of the womb. --[[User:TammyMoet|TammyMoet]] ([[User talk:TammyMoet|talk]]) 09:13, 6 March 2011 (UTC) Edit: Sorry, I meant an eggcup. The [[Menstrual cycle]] article says 10 - 80 ml. (Note to self: wake up before you start posting!) --[[User:TammyMoet|TammyMoet]] ([[User talk:TammyMoet|talk]]) 11:32, 6 March 2011 (UTC)
:It's [[water retention]]. Surprisingly little blood is lost during the menstrual period - about a teacup over the average 5 days. Most of what you see is [[endometrium]], or the lining of the womb. --[[User:TammyMoet|TammyMoet]] ([[User talk:TammyMoet|talk]]) 09:13, 6 March 2011 (UTC) Edit: Sorry, I meant an eggcup. The [[Menstrual cycle]] article says 10 - 80 ml. (Note to self: wake up before you start posting!) --[[User:TammyMoet|TammyMoet]] ([[User talk:TammyMoet|talk]]) 11:32, 6 March 2011 (UTC)

== Ability to dislocate shoulder ==

Hi,<br>
I can partially dislocate my shoulder (Enough to stick a finger in the gap created) and I was wondering, is there a reason for this ability, or is it just me?<br>
Thanks,<br>
[[User:Thomas888b|Thomas888b]] ([[User talk:Thomas888b|Say Hi]]) 14:47, 6 March 2011 (UTC)

:You may just have [[hypermobility]].--[[User:Aspro|Aspro]] ([[User talk:Aspro|talk]]) 15:36, 6 March 2011 (UTC)

Revision as of 15:43, 6 March 2011

Welcome to the science section
of the Wikipedia reference desk.
Select a section:
Want a faster answer?

Main page: Help searching Wikipedia

   

How can I get my question answered?

  • Select the section of the desk that best fits the general topic of your question (see the navigation column to the right).
  • Post your question to only one section, providing a short header that gives the topic of your question.
  • Type '~~~~' (that is, four tilde characters) at the end – this signs and dates your contribution so we know who wrote what and when.
  • Don't post personal contact information – it will be removed. Any answers will be provided here.
  • Please be as specific as possible, and include all relevant context – the usefulness of answers may depend on the context.
  • Note:
    • We don't answer (and may remove) questions that require medical diagnosis or legal advice.
    • We don't answer requests for opinions, predictions or debate.
    • We don't do your homework for you, though we'll help you past the stuck point.
    • We don't conduct original research or provide a free source of ideas, but we'll help you find information you need.



How do I answer a question?

Main page: Wikipedia:Reference desk/Guidelines

  • The best answers address the question directly, and back up facts with wikilinks and links to sources. Do not edit others' comments and do not give any medical or legal advice.
See also:


March 2

measuring and irrational numbers

I've been thinking about measuring and I was curious to know if it's OK to think of it this way:

Imagine we want to measure the length of an object.first we get 2m.and then with a more accurate measurement we get 2.1 ,2.12, 2.129 ,.... It's almost like an irrational number, because the decimals can't be predicted, and it also has an algorithm to be calculated more accurately (which is "measurement").so, which sentence is right?

1.the decimals never end and the object doesn't have a true, accurate length.

2. the decimals end somewhere and the object has an accurate, rational length. — Preceding unsigned comment added by Irrational number (talkcontribs) 13:00, 2 March 2011 (UTC)[reply]

Since 1983, (1) has been correct, except that stuff can only be measured with so much precision. See accuracy and precision. Before that, there was just one thing that was, by definition, exactly a metre in length.--Shantavira|feed me 13:25, 2 March 2011 (UTC)[reply]
I think you must be referring to the definition of the meter in terms of the speed of light, as described here Meter#Speed_of_light. However, I don't see how this definition of meter has anything to do with the claim that an objects don't have 'true, accurate' lengths. We could change the definition of a meter many of times, and this would never affect the material nature of e.g. the pencil on my desk. That is, the number used to quantify its length may change, but the pencil will not. SemanticMantis (talk) 15:33, 2 March 2011 (UTC)[reply]
Where I live, pencils get shorter.--Shantavira|feed me 16:41, 2 March 2011 (UTC)[reply]
I think this question is not about science, but about philosophy_of_physics, specifically regarding how to interpret the real numbers as a model for aspects of physical objects. I don't think there is a purely scientific basis for choosing to believe 1), 2), or other alternative interpretations. Perhaps someone else can shed more light on the philosophy involved here. SemanticMantis (talk) 15:33, 2 March 2011 (UTC)[reply]
Also, we should be careful with terminology 'accuracy' applies to measurements, not lengths. SemanticMantis (talk) 15:44, 2 March 2011 (UTC)[reply]
Having an unending decimal representation is not the same as being irrational – e.g. 1/3 = 0.333.. – so (1) and (2) don't cover all the possibilities even in an idealised situation. AndrewWTaylor (talk) 15:44, 2 March 2011 (UTC)[reply]
If the OP is talking about measuring real physical objects, then the answer is that a physical object does not have a "true", infinitely precise, length because the "length" of a physical object is a macroscopic concept and is not well defined at, say, picometre scales due to thermal motion, quantum effects etc. If the OP is talking about measuring ideal objects then the ratio of the lengths of two ideal objects may be either rational or irrational, depending on the objects involved. But I don't see how an irrational ratio is any less "true" or "accurate" than a rational ratio. Gandalf61 (talk) 15:45, 2 March 2011 (UTC)[reply]

I think it's better to ask my question this way:Is there any such thing as "the true length" or "the true value" for a certain object?Imean that we can do measurements more and more accurately, but is there any "end" for this process? — Preceding unsigned comment added by Irrational number (talkcontribs) 16:09, 2 March 2011 (UTC)[reply]

As Gandalf wrote, "length" for a physical object is a macroscopic concept that breaks down as you start measuring at the quantum level. In other words, you can only ever get to a certain precision (and you can always find a rational number expressing the length to that precision). On the other hand, if you talk about ideal mathematical objects, there are things that have irrational length (the diagonal of a square with sides of length 1) or even transcendental (the circumference of a circle with radius 1). --Stephan Schulz (talk) 19:35, 2 March 2011 (UTC)[reply]
In thought experiment only, couldn't the photon sphere around a black hole or neutron star be measured with arbitrary precision, by looking at the number of wavelengths of an arbitrarily high-frequency light source? Wnt (talk) 19:50, 2 March 2011 (UTC)[reply]
This makes me think of Coast#The_coastline_problem, not the same thing obviously, but similar. Vespine (talk) 00:41, 3 March 2011 (UTC)[reply]
The true length would be the limit. For example, in your 2.129... meter object, it will never get within 0.009 meters of 2.12, no matter how long you measure it. It's exactly 2 meters at the first step, but it's never within 0.1 meters of 2 meters after that. For any length except one, you could find a number of digits and a distance such that, after that many digits, you will never measure it within that distance of that length. — DanielLC 17:31, 3 March 2011 (UTC)[reply]
I am exactly one "Vespine" tall ;) Vespine (talk) 22:09, 3 March 2011 (UTC)[reply]

True or false? (about frogs)

"if you boil a pot of water and throw in a live frog that that frog will hop right out, saving his life to croak again another day (ha, ha)? If, on the other hand, you place a frog in a pot of cold water and turn the heat up slowly, that frog will stay in the pot." Quest09 (talk) 14:05, 2 March 2011 (UTC)[reply]

Talking from specific personal experience on this, if you boil a pot of water and throw a fish in, the shock will kill it instantly, or at least render it immobile. I believe the same would be true of frogs. --KägeTorä - (影虎) (TALK) 14:28, 2 March 2011 (UTC)[reply]
Isn't that animal cruelty? Quest09 (talk) 14:52, 2 March 2011 (UTC)[reply]
Probably. Depends on who you ask. We were cooking. I didn't enjoy doing it the first time, and I doubt I would do it again. --KägeTorä - (影虎) (TALK) 21:57, 2 March 2011 (UTC)[reply]
On one hand, the fish probably die quite quickly and I don't really believe fish "suffer" like we imagine anyway; on the other, it is completely "unnecessary" and only done because of the Japanese (sometimes perverted) obsession with "freshness". Completely no offense intended to KageTora, I personally love Japan and Japanese food, but throwing live fish into boiling water is not the worst of it. So I flip flop on the subject.Vespine (talk) 22:08, 2 March 2011 (UTC)[reply]
Exactly - I have seen far worse, which is why I said 'depends on who you ask'. Besides, people in western countries boil lobsters alive, and on rare occasions these lobsters are found to be still alive when served. --KägeTorä - (影虎) (TALK) 22:57, 2 March 2011 (UTC)[reply]
Can you provide some citation (not from PETA please) that a lobster can survive boiling water? Quest09 (talk) 02:12, 3 March 2011 (UTC)[reply]
Unfortunately I can only supply anecdotes in this case. Sorry about that. --KägeTorä - (影虎) (TALK) 02:23, 3 March 2011 (UTC)[reply]
We have a well-referenced article on the Boiling frog anecdote, which says: "According to contemporary biologists the premise of the story is not literally true; an actual frog submerged and gradually heated will jump out. However, some 19th century research experiments suggested that the underlying premise is true, provided the heating is gradual enough." WikiDao 14:30, 2 March 2011 (UTC)[reply]
Incredible that WK has an article even on that. Quest09 (talk) 14:52, 2 March 2011 (UTC)[reply]
Wikipedia has an article on everything! Of course we do!
Getting to the point of course, I doubt any animals at all (with a few exceptions) can withstand boiling water. Crimsonraptor(Contact me) Dumpster dive if you must 15:04, 2 March 2011 (UTC)[reply]
The question is if every animal will try to escape or if it will just numb and die. Quest09 (talk) 15:25, 2 March 2011 (UTC)[reply]
In truth, as someone who worked on getting that article into shape in 2009, we were spurred on to make it a good article (rather than just a bad anecdote and some biologists saying "not true") after being asked about it on the Ref Desk, if I recall. (I believe I was the one who really looked into the 19th century physiological literature for boiled frogs, which I happened to know was a very 19th century physiological thing to do with one's research time. But I may be mistaken.) It has been significantly edited since then, but bears the same hallmarks of the original '09 improvements. --Mr.98 (talk) 19:36, 2 March 2011 (UTC)[reply]
What are you talking about? According to [1] you didn't contributed anything to that article. Unless, oh no, you are a sock-puppet? Right? Quest09 (talk) 02:33, 3 March 2011 (UTC)[reply]
I do have multiple accounts (which I have never been shy about), which is not disallowed and not sock-puppetry. (I am not using them to evade bans or anything inappropriate. None of my accounts are in bad standing. One is even an administrator! Although in fairness I never use that one for much anymore, and have not used its admin powers in years.) But in any case, this was when I edited as an IP, before I registered this particular user name. It turns out it was in 2007, not 2009. Look for some major overhauls by a certain 24.x.x.x, some others by 98.x.x.x. --Mr.98 (talk) 20:37, 3 March 2011 (UTC)[reply]

dimensions

Could matter, motion, thought, energy, light, etc. possibly be dimensions? For example 4-d spacetime could exist and not result in matter or thought. —Preceding unsigned comment added by 165.212.189.187 (talk) 16:19, 2 March 2011 (UTC)[reply]

If you redefine what "dimension" means, then anything can be a dimension. As for the standard definition of "dimension", they are not dimensions. -- kainaw 16:20, 2 March 2011 (UTC)[reply]
In physics, we use generalized coordinates to describe the complete state of a system. Anything can be defined by a "coordinate" - but only a few are useful coordinates that help us simplify a description of an observed physical system. In addition to your typical "x,y,z" coordinates, physicists often use velocity, angular displacement, and energy as a "coordinate" of a system. Typically, we seek to use a linear transform to convert from an orthonormal coordinate scheme (like a cartesian grid or a polar coordinate system) into a generalized coordinate space; this makes the math much simpler. It is sometimes, but not always, possible to obtain "energy" via a differentiation of a spatial coordinate; (for example, kinetic energy is related velocity squared - which is a nonlinear operation on the first derivative of a spatial coordinate). So, "energy" is rarely a useful coordinate in most physical treatments.

But you must know the mass to get the actual energy, right? —Preceding unsigned comment added by 165.212.189.187 (talk) 20:10, 4 March 2011 (UTC)[reply]

For a concrete example: consider a robot arm. Its hand has an "x,y,z" position; but this is not the easiest set of coordinates to "control the robot." Instead, you want to design a coordinate-system based on the position-values and velocity-values for each "joint" or "elbow" in the robot-arm, so that you can send motor control commands. This approach is called Forward kinematics and is an elementary part of robot theory. More advanced robot control will use other coordinates to design an objective function: for example, you could have a robot-arm who is aware of a coordinate called the "value" of a position, and thus prefers to move to "high-value" coordinates (independent of their x,y,z coordinate). You can thus introduce a new "coordinate" and call it anything you like. This "coordinate" or "dimension" might be the pixel-value at some location in a computer vision camera; or it could be the result of a complex algorithm, or any other thing you want. The reason I bring this up is because advanced robotic theory is mostly concerned with the matrix-mathematics needed to find optimal solutions in high-dimensional spaces. Nimur (talk) 16:37, 2 March 2011 (UTC)[reply]

OK, then I guess my question is can current physics describe information and thought the way it describes light and heat? —Preceding unsigned comment added by 165.212.189.187 (talk) 16:58, 2 March 2011 (UTC)[reply]

No. See information theory and artificial intelligence for the overview of how "information" and "thought" are modeled scientifically using current theories and technology. Nimur (talk) 17:04, 2 March 2011 (UTC)[reply]

But a true "theory of everything" would have to describe them, right? —Preceding unsigned comment added by 165.212.189.187 (talk) 18:13, 2 March 2011 (UTC)[reply]

I should just mention Kaluza-Klein theory, M-theory, string theory and so on where the modeling of matter, energy, and light is concerned. Typically information is thought of in terms of other quantities - when you measure the position of an electron and such - I don't know if there's a way to quantify it in such a theory independently of specific forces and particles. But thought, or the "when's dinner?" kind of information, is really not well understood and quite macroscopic (even to the point that no specific part of the brain is certain to affect it when damaged). I'm sure that there are some chip makers who daydream about shrinking their transistors down to the scale of a compactified dimension and using them to operate artificial intelligence, but (probably luckily) this is not on the drawing board yet) Wnt (talk) 18:30, 2 March 2011 (UTC)[reply]
There is a theory that all information is retained in the universe in just two dimensions and that any concept we have of a third dimension is nothing more than how our brains perceive the two dimensional information. I haven't read anything on this theory in a few years and I'm certain it has been completely discredited by now. This is an old paper on it by someone who knows what he is talking about. -- kainaw 19:04, 2 March 2011 (UTC)[reply]
The Holographic principle seems to a mainstream (if highly theoretical) idea of this type. - Jarry1250 [Who? Discuss.] 19:32, 2 March 2011 (UTC)[reply]

The holographic principal would help explain my problem with density. —Preceding unsigned comment added by 165.212.189.187 (talk) 21:07, 2 March 2011 (UTC)[reply]

I just want to point out that when physicists say theory of everything they mean a theory of all matter and forces, nothing else. Dauto (talk) 19:30, 2 March 2011 (UTC)[reply]
(ec) Well, and the reductionist, materialist world-view is predicated on the belief that all complex processes (including sentient thought, consciousness, reason, and sophisticated intelligent life) are fundamentally nothing more than extremely large numbers of simply-interacting objects that behave according to fundamental physics. This is a philosophical position - it is neither correct nor incorrect; it falls outside the realm of "falsifiable" or "provable." But if you hold this world-view, then intelligent thought and complex information storage can be completely described as the ensemble definition of all applicable rules and the state of a complex material representation of information. For example, the bits inside a computer are represented by voltage on microscopic capacitors; those voltages exist because of electrons in a semiconductor-material that obey fundamental rules of electrodynamics. It doesn't matter how complex the computer is behaving - the information it contains is still governed by basic rules of elementary physics. Unfortunately, we are not as close to understanding the way that psychology and memory works in living creatures; so it is much harder to apply a materialist/reductionist worldview as a description of human intelligence. Nimur (talk) 19:54, 2 March 2011 (UTC)[reply]
The fact that a proposition may be neither falsifiable nor provable, does not imply that it is neither correct nor incorrect. It only says we may have trouble finding out whether it is correct or incorrect. --Trovatore (talk) 21:17, 2 March 2011 (UTC)[reply]
"it falls outside the realm of falsifiable or provable." No, it is falsifiable or provable, if e.g. whole brain emulation is possible then it follows that thought, consciousness, reason, and sophisticated intelligent life, are indeed nothing more than extremely large numbers of simply-interacting objects that behave according to fundamental physics. 213.49.110.122 (talk) 06:00, 3 March 2011 (UTC)[reply]
There's no way to tell whether an emulator is actually experiencing anything subjectively, whether it has qualia. (For that matter, it's hard to say how I can tell whether anyone but me has qualia. But I do; I guarantee you.) --Trovatore (talk) 22:07, 3 March 2011 (UTC)[reply]
Thought is something that occurs inside brains, so it's a question for biology, not physics. thx1138 (talk) 23:26, 2 March 2011 (UTC)[reply]
You could possibly explain thought with quantum mechanics, as the collapse of the wavefunction of one electron in a nerve cell by another (Or maybe in the manner that quantum computing works). After all, quantum mechanics is the reason our free will is preserved, and thus it might be the reason for free will itself. ManishEarthTalkStalk 09:04, 3 March 2011 (UTC)[reply]
While it's certainly possible that's the case, there's no currently no evidence to support it. thx1138 (talk) 16:06, 3 March 2011 (UTC)[reply]
Anyone interested in the possible relationship of quantum mechanics to conscious thought should read Roger Penrose's The Emperor's New Mind. As both those articles indicate, the theory hasn't gained much traction. Matt Deres (talk) 18:18, 3 March 2011 (UTC)[reply]
If time can go backwerd , and it can , it can go proprate to matter,or any thing we want to relate to , matter too will be dimensions . thanks water nosfim —Preceding unsigned comment added by 212.199.175.104 (talk) 19:49, 2 March 2011 (UTC)[reply]

Maxwellian conflict

Paradigm shift talks about the contest between Maxwellian electromagnetism and Einsteinian relativistic theory. However my understanding was that prior to Einstein, Maxwell's equations were one of the few bits of physics which did already Lorentz transform correctly. Certainly the equations I was taught at university to be Maxwell's equations, are the same as those which are pulled out by considering the force resulting from the EM-Stress energy tensor. So where was there conflict between the two? —Preceding unsigned comment added by 129.67.37.227 (talk) 19:58, 2 March 2011 (UTC)[reply]

That does seem peculiar. History of special relativity#Lorentz's theory of electrons and onward covers the history in better detail. Wnt (talk) 20:41, 2 March 2011 (UTC)[reply]
(ec) I have never read The Structure of Scientific Revolutions. Despite its wide acclaim, I have preferred to actually read science books, instead of meta-debate about science. It is my opinion that there isn't a dramatic "transition between the Maxwellian electromagnetic worldview and the Einsteinian Relativistic worldview." Our Paradigm shift article attributes that belief to Kuhn - i.e., implies that Kuhn considered this a "dramatic shift" - so what we need to do is determine if our summary is consistent with Kuhn's writing. Whether it is applicable is subject to your own opinion, and whether you agree with Kuhn.
You can read Maxwell's own work: On Physical Lines of Force and A Dynamical Theory of the Electromagnetic Field are the most relevant. (Bear in mind that his notation differs from the most popular modern notation, but the physics is identical). Our articles link to full text versions of these historic works. You can also read On the Electrodynamics of Moving Bodies by Einstein; and decide for yourself whether there is a "dramatic" paradigm shift, or a "gradual, constructive evolution." I will also point out that Hendrik Lorentz invented and started using his his transform without ever before ever attributing its mechanism to relativity. In a sense, it was a "subtle modification" that built on earlier theory to improve accuracy. Nimur (talk) 20:53, 2 March 2011 (UTC)[reply]
For a guy who coaches his opinions in a love of science, it is very unscientific to comment on things you have not read! Just read the book already, it's a quick read, you'll at least know whether you actually agree with it or not... "science books" are usually quite poor sources for their history. You can find "history books" for that. Some written by that know-nothing, Thomas Kuhn! --Mr.98 (talk) 03:13, 3 March 2011 (UTC)[reply]
The article has been poorly wiki-linked. What's important is the "electromagnetic worldview," which is not the same thing as Maxwell's equations. Luminiferous aether would be a better link. It's not the equations, per se, which actually work fine. It's the intuitive way in which the world was thus thought to work. Incidentally if you want to get a sense for the actual historical progression of things, it gets into things that are far more detailed than your standard physics textbook, which really only describes the history so far as it contributes to the present-day theories. The actual history of these things is always more complicated. If you're looking for a great, one-stop-shop history of modern physics (which discusses but is not heavily burdened by theoretical models about scientific change), I would recommend Helge Kragh's Quantum Generations, which is especially good for people who are interested in the actual physics, as well as its context, etc. Kragh's book (unlike most "popular" histories of physics) has the virtues of being both accurate and interesting; it is the sort of thing that actual historians of science read when learning this stuff themselves. --Mr.98 (talk) 03:13, 3 March 2011 (UTC)[reply]
And note that I write this not as someone who is a real "Kuhnian." Kuhn's arguments are that shifts between "paradigms" (a big and vague category that includes theories, intuitive models of the world, pedagogy, experimental expectations, etc.) is as massive and discrete as a Gestalt shift. There are perhaps a few candidate incidents for the "old" worldview and the "new" worldview being Gestalt-like, and the EM worldview/relativity is perhaps one of them (Copernican vs. geocentric might be another). But it's unclear if that's really how science works most of the time, and Kuhn himself, for all of his virtues, did not really do the historical work to actually find these shifts. The Kragh book shows how complicated actual shifts of this nature can be; it's hard (impossible?) to come up with an historical theory of how scientific change works that takes into account the complexity of real-life examples, much less real-life examples for all of what can be considered "science." On the whole, philosophers and historians of science have moved away from attempts to do this sort of thing, and have been more purely descriptive (along Feyerabendian lines, albeit without his political motivations) in their actual scholarship. --Mr.98 (talk) 13:57, 3 March 2011 (UTC)[reply]
Kuhn does not claim that this is how science works most of the time. Paradigm shifts are rare events. --Srleffler (talk) 17:31, 5 March 2011 (UTC)[reply]
You can have a paradigm shift without the equations changing. Maxwell constructed a very complicated idea of how his equations might work within the bounds of Newtonian physics. He didn't have it extending to everything and he didn't have time going at different rates or energy and mass being the same. Dmcq (talk) 14:21, 3 March 2011 (UTC)[reply]
And in fact, in a typical Kuhnian description, most of the equations would stay the same, but the meaning behind them would change. A very easy concrete example of this is that a Lorentz contraction meant a very different thing to Lorentz than it did to Einstein. To Lorentz it had to do with how the aether worked; for Einstein, it was a statement about the implications of relative frames of reference. Boyle's Law still holds even though we have a very different understanding of gases than Boyle himself did. --Mr.98 (talk) 14:47, 3 March 2011 (UTC)[reply]

Historical scientific measurements

I have trawled wiki and google for a while to no avail. Does anyone know where I can find a plot of historical measurements of the electron charge and Hubble's constant. I.e. a list of data or plot of best estimates against when they were made, in order to show the effect of which Feynmann spoke in his cargo cult science talk. —Preceding unsigned comment added by 129.67.37.227 (talk) 23:50, 2 March 2011 (UTC)[reply]

Here's one for Hubble's constant from Harvard (about halfway down). Clarityfiend (talk) 03:10, 3 March 2011 (UTC)[reply]
The charge on an electron is formally called the elementary charge. According to the Oil-drop_experiment article, Robert Millikan's initial measurement wass about 1% lower than the currently accepted value. CS Miller (talk) 13:00, 3 March 2011 (UTC)[reply]
Indeed, but Feynmann reports that nonetheless sociological factors encourage scientists to misreport results leading to a kind ofof exponential decay toward the accepted value. Thank you for the Hubble data. —Preceding unsigned comment added by 129.67.37.227 (talk) 14:08, 3 March 2011 (UTC)[reply]
No trubble at all. Clarityfiend (talk) 01:11, 4 March 2011 (UTC)[reply]

I couldn't find the plot of measured electron charge over time, though I've seen it before and it's a very good illustration of how previous experiments bias the results of new one. Over the years, new experiments eke further and further away from millikans result. There were no random spread distribution around the true value, as you would have expected if only measurement error was to blame. for a similar situation, here's a plot of the measured speed of light:[2] Notice how the true value is outside the reported margins of errors of several experiments. EverGreg (talk) 09:09, 4 March 2011 (UTC)[reply]


March 3

observing the observer

How would THE observer explain another observer, such as humans or amoeba, which have free will and the ability to create endless entropy through thought and interaction with/unexpected manipulation of its known universe? Does an observer only have vision, hearing, smell, feeling(physical), taste?

These kinds of question essentially boil down to Ontology and even Epistemology, which are very broad and complex subjects. People have been contemplating them for thousands of years, so I don't think you'll get a quick and easy answer. I would say however that gathering knowledge does not have to be restricted to the physical senses, otherwise how would we know that atoms exist. Vespine (talk) 03:08, 3 March 2011 (UTC)[reply]

B/C we use our senses to interpret the output of the apparatus that was made by us (with our hands).

In which case you again come back to epistemology. It was a rhetorical question. Vespine (talk) 22:05, 3 March 2011 (UTC)[reply]

Actually you misinterpreted the question. Like if the observer were observing 4-d space with some matter and some energy in it, and he was doing this for a period of time, when a pteradactyl flies by the matter and interacts with the energy, how would the observer explain this and his opinion of the matter and energy from that point on. Using the laws of physics, only of course. wouldnt he have to include that event mathmatically for every subsequent observation?

In which case sorry but I don't really understand what you are trying to ask.. What do you mean by how would they explain it? Do you mean how would they model the interaction? Or how would they explain the forces involved? Or how would they predict the outcome? Or something else? The initial question was something about using your senses, while ultimately anything you can perceive or conceive is mediated by the senses, no matter what instruments or techniques you use, that doesn't mean nothing is real and nothing can be known or explained. Vespine (talk) 03:24, 4 March 2011 (UTC)[reply]

Yes. yes. no. no. I meant in the article "observer" it talks about the observer being the reference frame -four dimensions of spacetime. 'Note that these uses differ significantly from the ordinary English meaning of "observer". Reference frames are inherently nonlocal constructs, covering all of space and time or a nontrivial part of it; thus it does not make sense to speak of an observer (in the special relativistic sense) having a location. Also, an inertial observer cannot accelerate at a later time, nor can an accelerating observer stop accelerating.' But that alone does not ensure the ability to percieve the universe, just part of it (empty space). it gives matter a place to be, but not matter or energy itself.

Personally, I think the whole concept of observation is a pointless fudge of a concept - everything in the universe is observing everything else, albeit indirectly or otherwise. There is no yes or no answer to whether or not some process is being observed or have been observed. It's all about "stage fright", is the audience big enough. In double slit electron experiment, the apparatus is observing the electron's double path, even if the exprimentor is not. I think it's related to the Butterfly effect, the further the effects of a proccess become magnified, the less stable the super position becomes. (the bigger the "stage fright"). Plasmic Physics (talk) 04:49, 4 March 2011 (UTC)[reply]

That will make it easy for me to disregard your observation.

How is that? I think you're confusing observation with deduction. Plasmic Physics (talk) 21:25, 4 March 2011 (UTC)[reply]

Kind of like how you confuse have with has. You are missing the point I am not talking about AN observer, I am talking about the objective frame of reference like you are talking about, du, the apparatus - the observer "'Note that these uses differ significantly from the ordinary English meaning of "observer". Reference frames are inherently nonlocal constructs, covering all of space and time or a nontrivial part of it; thus it does not make sense to speak of an observer (in the special relativistic sense) having a location. Also, an inertial observer cannot accelerate at a later time, nor can an accelerating observer stop accelerating.'

I know, I was just making a side note. Not sure about your logic though. A reference frame is completely arbitrary, with an assigned coordinate relative to an observer and event. Plasmic Physics (talk) 02:04, 5 March 2011 (UTC)[reply]

OK in my example above, wouldn't the "laws" of physics all have to contain the effect of the flyby forever? Every measurement would discover the anomaly and have to explain it or on the contrary incorporate it as part of the laws. If you didn't observe the flyby you would have no idea that it was a pteradactyl. We know that the flyby has is happened(ing) at least once because we are the ones flying by. Why couldnt it have happened any number of times before.

If I wasn't confused before, I am now. I leave it to someone else to interpret. Plasmic Physics (talk) 12:35, 5 March 2011 (UTC)[reply]

dividing some random units

What happens when I divide two random physical units? To me, it is clear that km/h is speed. But, what about height/temperature of a body? May I name the resulting unit towards me? 212.169.189.17 (talk) 03:05, 3 March 2011 (UTC)[reply]

It's adoption not discovery which gives units their names. You haven't really "discovered" anything new or novel. Celsius is named after a person, but he didn't "discover" temperature, he invented a good and reliable way to measure it, so people used it, he didn't "force" people to use it or impose some sort of patent to measuring temperature that everyone had to use, even now lots of people prefer Fahrenheit. It's not enough to simply stick 2 existing units together, you have to "offer" something new that people will want to adopt. Vespine (talk) 03:27, 3 March 2011 (UTC)[reply]
You're welcome to name a derived unit after yourself, but as Vespine notes, no one is likely to follow your lead without compelling reason to do so. The General Conference on Weights and Measures (CGPM) is responsible for officially adopting names and prefixes for the units which form the International System of Units (SI, or the 'metric system'). If you can sell the CGPM on your new unit, you'll be set—otherwise, this is an unlikely route for you to achieve immortality. TenOfAllTrades(talk) 03:35, 3 March 2011 (UTC)[reply]
Dimensional analysis shows what happens. You need to use absolute temperature to get anything physically meaningful. Dmcq (talk) 04:11, 3 March 2011 (UTC)[reply]
If I divide my absolute temperature (310 K) by my height in cm (181) I don't believe I get anything meaningful. What does 1,71270718 K/cm mean? The calculation are, at the first sight, right. Quest09 (talk) 15:32, 3 March 2011 (UTC)[reply]
How compound units like that become useful is through data correlation. For this example, assume I had tons of patient data and I'm looking at some outcome. I find that the outcome is positively correlated to the patient's temperature and negatively correlated to the patients height. I would then derive a formula that multiplies some constant by the temperature and divides by the height. Then, I'd have a result in temp/height units that would have meaning for that particular outcome. So, trying to make a compound unit without reason simply isn't reasonable. Deriving it as part of an observation is reasonable. -- kainaw 15:43, 3 March 2011 (UTC)[reply]
The PDF by Vignaux, G.A. (1991). "Dimensional analysis in data modelling", linked to in the Buckingham π theorem article, discusses that point. 92.15.15.174 (talk) 21:25, 3 March 2011 (UTC)[reply]
At one time it amused me to rank colleges by "books per acre," as opposed to more commonly cited stats like "student-teacher ratio." A large midwestern (US) university needed a humongous library to equal a compact downtown college in my index. Thus Purdue University (2.5 million books, 15,000 acres) has a ratio of 166, slightly higher than Shimer College, ratio 120 (20,000 books, 166 acres. Michigan State University ranks higher with a ratio of 490 (4.9 million books, 10,000 acres. This ratio addresses the issue of whether one would prefer to read or wander around. Edison (talk) 05:11, 4 March 2011 (UTC)[reply]
Thats funny, that is not the information I used before I would wander around my campus, even if I has such info. —Preceding unsigned comment added by 165.212.189.187 (talk) 13:46, 4 March 2011 (UTC)[reply]
One of my instructors once mentioned how you can get in trouble with blind dimensional analysis. Consider a sailboat of length L moving at speed V. Then the age of the captain is T=L/V. Short Brigade Harvester Boris (talk) 08:16, 5 March 2011 (UTC)[reply]
This is just the wrong way to think about it. You divide physical units to get a new unit to reflect that you divide physical quantities to get a new, meaningful quantity. By starting out thinking about dividing units, you're doing something that has no meaning at all. You have to think why you are dividing height by temperature, and what the result might mean.--Srleffler (talk) 17:49, 5 March 2011 (UTC)[reply]
The "Level of measurement" http://people.math.sfu.ca/~cschwarz/Stat-301/Handouts/node5.html also gives you some clues as to what can be properly divided into what, as does their Intensive and extensive properties. 92.15.6.227 (talk) 19:33, 5 March 2011 (UTC)[reply]

ie (Internet Explorer)

title expanded by CS Miller (talk) 12:53, 3 March 2011 (UTC) [reply]

how can i make firefox or ie save my address when i make a order. it wont do it — Preceding unsigned comment added by Wdk789 (talkcontribs) 05:51, 3 March 2011 (UTC)[reply]

You'll need to provide more information - but do so at WP:Reference desk/Computing for better results! Wnt (talk) 05:56, 3 March 2011 (UTC)[reply]
Do you want Firefox/Internet Explorer to automatically fill out your address in web forms? Firefox stores the username/password fields if enabled ( Tools / Options / Security ). It can also guess what to put in the form, for example, if a form has a field called "username", then firefox will examine the value of all the "username" fields that you've previously filled out, and use this for the auto-completion values. As WNT said, you'd be far better of asking at the computing desk for more details. CS Miller (talk) 12:53, 3 March 2011 (UTC)[reply]

IT IS ENABLED BUT ITS not working sry caps — Preceding unsigned comment added by Wdk789 (talkcontribs) 11:15, 4 March 2011 (UTC)[reply]

Sydney Basin

Why is Sydney Basin called Sydney Basin? 220.233.83.26 (talk) 06:25, 3 March 2011 (UTC)[reply]

Have you tried reading Sydney Basin? Lots of information there. Dolphin (t) 06:33, 3 March 2011 (UTC)[reply]

yer i have now thanks! but just quickly how was the basin formed in like a dead easy way that even i can understand. i anyone could help me with this i would be really appreciative. 220.233.83.26 (talk) 06:48, 3 March 2011 (UTC)[reply]

alright thanks guys for answering me... :/ ^^ 220.233.83.26 (talk) 09:18, 3 March 2011 (UTC)[reply]

The Sydney Basin is an example of a foreland basin. Foreland basins area created by the loading of the leading edge of a tectonic plate by a thrust belt. This loading cause downflexing and the accommodation space is then filled with sediment. The loading also causes a flexural forebulge to develop, which migrates with time into the foreland as does the foreland basin. A typical sequence would be - continental to shallow water sediments derived from the forebulge followed by progressively deeper water marine sediments (often called a flysch sequence) finally filled in by coarse material (conglomerates and sands derived from the mountain belt) deposited into progressively shallower water before becoming continental again (often referred to as a molasse sequence). The sequence described for the Sydney Basin matches this pretty well. Mikenorton (talk) 10:07, 3 March 2011 (UTC)[reply]

Oral Polio vaccine controversies

A statement issued by the Second World Social Forum on Health condemned the "World Health Organisation's lack of transparency in acknowledging the failure of the Global Polio Eradication Initiative strategy and instead...using monovalent OPV, an untested vaccine, without informed consent." According to this document, mOPV1 is not a new vaccine. "Except that it does not contain types 2 and 3, mOPV1 is in all other respects similar to tOPV...monovalent oral polio vaccines for all the 3 types of polioviruses were used extensively in the early days of polio vaccination in the late 1950s and early 1960s". Is there any merit in the statement of World Social Forum on Health? Did they withdraw it later? There have been condemnations about the trials of bivalent polio vaccines in India as well. Is the situation same? A WHO document mentions "In 2009, a clinical trial was conducted in India to compare the rates of seroconversion to WPV1 and WPV3 generated by a bOPV with that of the respective monovalent OPVs (mOPV) and trivalent OPV (tOPV)". But the Government of India, while introducing the vaccine in 2009, claims that "The vaccine will be new for India but it is an established vaccine with many developed countries already using it".14.139.128.15 (talk) 07:13, 3 March 2011 (UTC)[reply]

The “negative aspects” mentioned in the Second World Social Forum on Health highlight the fact that when wild polio is 'almost' eliminated from a geographic area, then most cases of acute flaccid paralysis arises from the live attenuated polio vaccine viruses reverting back to their 'wild' and dangerous form. As different companies often come out with their own version of existing vaccines, I assume that the untested OPV mentioned, is new product and thus still untested an OPV(as of 2007) rather than one of the older ones. Ideally, prophylactic measures employing killed injectable polio virus (which can't revert back) would be therefore more advantageous to mope up the last remnants of polio in a nearly polio free area but unfortunately these countries are denied these more expensive injectable vaccines due to cost augments. Thus the local people start to resist getting any more of their children vaccinated for this disease and the local health providers get the blame for poor vaccine coverage and the inevitable return of circulating polio. India is now facing the same situation where it would make more health sense to switch over to injectable killed viruses. It all come down to money. That's it I think -in a nut shell.--Aspro (talk) 16:34, 3 March 2011 (UTC)[reply]
Ah! I caould have saved my breath as an article in NEJM explains it even better.The Bumpy Road to Polio Eradication --Aspro (talk) 16:58, 3 March 2011 (UTC)[reply]
Thank you very for for your reply and also for showing me the NEJM article. In response to a mail of mine, the author, John F. Modlin, said that "The statement by the World Social Forum on Health is frankly disingenuous and misleading. The monovalent vaccines used in India today are derived from the same strains used 50 years ago to control polio in many (at the time) industrialized countries. I would not give this statement any credence." 14.139.128.15 (talk) 13:45, 5 March 2011 (UTC)[reply]
Whoa whoa whoa! If that's the case, then why do countries like the UK still use oral polio vaccine? Last I checked, polio was pretty much eliminated here except for unvaccinated people changing recently-vaccinated babies' nappies. I know there was something about new mothers being vaccinated to avoid that, but why not switch to injectable killed virus? 82.24.248.137 (talk) 21:45, 4 March 2011 (UTC)[reply]
Is OPV still used in UK? I read that "Since September 2004, the UK vaccination programme has used inactivated vaccine (IPV)" and "JCVI has recommended that a switch can be made from live oral polio vaccine (OPV), which provides good community protection, to inactivated polio vaccine (IPV), which provides effective individual protection"14.139.128.15 (talk) 13:45, 5 March 2011 (UTC)[reply]
I have checked on the BNF just to make sure of the situation now, as these things often change. A live oral vaccine is still licensed and available to control an outbreak, should there be one and can thus can be supplied in one and ten dose containers. So NOT used but can be. Routine immunisation still uses as of 2004 (as the two above references state) inactivated forms and usually administered as one of the polyvalent vaccines which includes diphtheria and/or tetanus. I wouldn't give undue wait to any criticism any doctor levels at an organisation or forum without knowing just how deeply they are involved in creating the vaccination policy being criticised or promoting the pharmaceutical company that manufactures the product being used or both. If, instead of answering the criticisms, they come back with flat out contradictions (a new vaccine might be derived from the same strain isolated 50 years ago but how does that make the new product proven - it's a non sequitur ), ect... Well! I leave you to judge.--Aspro (talk) 15:34, 5 March 2011 (UTC)[reply]

Immbolization and chest pain

A patient with history of immbolization also suffers from chest pain. How closely does the patient's past history of immbolization affect the patient, who also has recurrent chest pains? aniketnik (talk) 08:07, 3 March 2011 (UTC)[reply]

Whoa, sorry I misread the question!!! Ignore response below. I assume you mean embolisation
What caused the immobilisation? was it muscular? neurological? trauma? Does the term "past history" mean that the patient is no longer immobile? What is the character of the chest pain? I think a more focussed question would be helpful - assuming this is not seeking medical advice. Richard Avery (talk) 08:29, 3 March 2011 (UTC)[reply]
Is this a homework question? Ariel. (talk) 09:58, 3 March 2011 (UTC)[reply]

The patient has a past medical history of immobilisation because of a motor vehicle accident that happened many years ago. Following the accident the patient was in a knee immobilizer for a short span of time. The musculoskeletal system of the patient is now functioning properly. So the patient is now no longer immobile. The patient is an obese male and being obese the patient has recurrent sharp chest pain, which occurs at regular intervals. So does the past history of immobilization followed by obesity, is it going to one of the major contributing factor to the patient's physical condition? Is the past medical history of immobilization affecting the patient's health and also in one way or other responsible for the frequent and recurrent chest pains? — Preceding unsigned comment added by Aniketnik (talkcontribs) 04:31, 4 March 2011 (UTC)[reply]

This reference desk can not answer these types of medical questions, sorry. Does the patient have a doctor? Ariel. (talk) 07:08, 4 March 2011 (UTC)[reply]

Allergy to Meperidine

The patient has an allergy to Meperidine drug. What effect does Meperidine have on a patient who has an allergy to Meperidine? aniketnik (talk) 08:10, 3 March 2011 (UTC)[reply]

This question is borderline on if we can answer it. On one hand it's a request for information, not diagnoses. But on the other it sounds like you would make a medical decision based on what we would say, and that would be a problem. I can point you to Meperidine which has some information on this drug (although nothing about allergy). And also suggest that since this is a painkilling drug, perhaps pick a different one. Ariel. (talk) 09:56, 3 March 2011 (UTC)[reply]
Also: Is this a homework question? Ariel. (talk) 09:58, 3 March 2011 (UTC)[reply]
Most likely. SmartSE (talk) 13:30, 3 March 2011 (UTC)[reply]
Googling "Meperidine allergy" finds a paper saying it causes urticaria. SmartSE (talk) 13:30, 3 March 2011 (UTC)[reply]

Gravity

Does the gravitational effects of an object increase with it's energy (and hence mass)? 74.15.137.130 (talk) 08:44, 3 March 2011 (UTC)[reply]

Yes. But since energy is relative things can get pretty complicated. Ariel. (talk) 09:54, 3 March 2011 (UTC)[reply]
Great, thanks. 74.15.137.130 (talk) 16:15, 3 March 2011 (UTC)[reply]

Why is the shannon diversity denoted H'? Our article Entropy_(information_theory) uses H for the same quantity. Enthalpy is also denoted by H, but physical entropy uses S. So, is there some rationale or historical reason for using H' to denote the Shannon-wiener index? Thanks, SemanticMantis (talk) 15:57, 3 March 2011 (UTC)[reply]

The rationale is that it is calculated using the same formula. The only difference is that the values that get plugged into the formula are population fractions rather than probabilities. Looie496 (talk) 17:50, 3 March 2011 (UTC)[reply]
The question is since, as you pointed out, this index uses the same formula used for entropy, why don't they use the letter S to represent it? Dauto (talk) 19:15, 3 March 2011 (UTC)[reply]
Ok, I just dug into Shannon (1948). He uses H, and says it is the same as Boltzmann's H from "Boltzmann's famous H theorem." Am I correct to assume this must have been Boltzmann's entropy? That is, perhaps Boltzmann originally used H, but later changed to S. In any case the H' notation seems spurious, I may change our article to H unless I find any good reason not to. As it stands, using H' in some cases and H in others makes a distinction without a difference. SemanticMantis (talk) 19:31, 3 March 2011 (UTC)[reply]
Mathematicians in general don't have much patience with worries about what letters are used to represent things. In fact, one of the greatest challenges in teaching mathematics is getting students to see through the letters and symbols to the conceptual entities they are supposed to represent. Looie496 (talk) 01:23, 4 March 2011 (UTC)[reply]
Indeed. However, compact, efficient notation is still useful and desirable. Also, I'm specifically talking about the use and notation of the shannon index in biology, not as a topic in pure math. "Seeing through letters and symbols" is not something an undergrad bio major is accustomed to, so why make it more difficult than we have to? SemanticMantis (talk) 15:19, 4 March 2011 (UTC)[reply]

what is magnetic polarity in a electric current

sorry for the strange wording but what is magnetic polarity in a electric current — Preceding unsigned comment added by Lufc88 (talkcontribs) 19:13, 3 March 2011 (UTC)[reply]

See Right-hand rule.
Prediction of direction of field (B), given that the current I flows in the direction of the thumb.
. If your right hand were encircling a wire carrying current, with the thumb pointing in the direction conventional current (positive to negative) flows, then the fingers point in the direction of the magnetic B field. The arrow on the B field corresponds to the North pole of a magnet. The magnetic field corresponds to a series of concentric circles, and the field is stronger near the conductor. The arrangement of the conductor (one wire, a solenoid, a solenoid around a ferromagneticmaterial) strongly affects the shape and strength of the magnetic field. (Do not actually put your hand around a bare wire carrying current, since you might get electrocuted! It is just a "thought experiment.") Edison (talk)
Just chiming in to link the pictures at solenoid. Loosely, when the wire is straight, the magnetic field lines are circular, but wrapping the wire in a coil produces a magnetic field with straight lines inside the solenoid. SemanticMantis (talk) 21:17, 3 March 2011 (UTC)[reply]

STS-133 and auroras

Hi. The Sun is currently in an active phase, and some sunspots as well as coronal holes are present and capable of producing both solar wind and strong solar flares. My question is, do the potential X-class solar flares pose a risk to astronauts currently in space? Also, are the SDO and other instruments capable of monitoring sunspots behind or to the side of the Sun? Thanks. ~AH1(TCU) 19:46, 3 March 2011 (UTC)[reply]

This isn't a complete answer at all, but this link discusses the Forbush decrease, in which a coronal mass ejection results in less radiation absorbed by astronauts who happen to be up there at the time. Comet Tuttle (talk) 21:04, 3 March 2011 (UTC)[reply]
In Earth orbit the Magnetosphere attenuates the particle flux down. This NASA article explains the potential X class danger to the station. Sickening Solar Flares . As the article mentions, the aluminium hull further reduces the danger. The impact though, produces a shower of secondary particles and so a plastic shield (because it is made of lots of light atoms) has been added to reduce these secondaries.--Aspro (talk) 22:04, 3 March 2011 (UTC)[reply]

One eye

What's that syndrome called where a child is born with one centre eye? Albacore (talk) 22:21, 3 March 2011 (UTC)[reply]

Cyclopia. thx1138 (talk) 22:26, 3 March 2011 (UTC)[reply]
Cyclopia DMacks (talk) 22:26, 3 March 2011 (UTC)[reply]
And don't forget the South Park episode where Kenny was pictured with "one brown eye". StuRat (talk) 02:43, 4 March 2011 (UTC) :-) [reply]

Is the Big Bang, a fact?

Topic says it all. ScienceApe (talk) 23:42, 3 March 2011 (UTC)[reply]

Let us know what you find out after you've read Big Bang. TenOfAllTrades(talk) 23:56, 3 March 2011 (UTC)[reply]
The Big Bang (the event) is better classified, per the article lede, as "the event which led to the formation of the universe, according to the prevailing cosmological theory of the universe's early development (known as the Big Bang theory)" than as a "fact". We also note that the Big Bang theory incorporates some of the best-ever matches between theoretical predictions and observed values in all of science (the Cosmic Microwave Background).
Alternately, what do you mean by "fact"? — Lomn 00:01, 4 March 2011 (UTC)[reply]
I'm just debating with an annoying religious guy, and I want to make sure I'm correct if I say the Big Bang is a fact. ScienceApe (talk) 00:03, 4 March 2011 (UTC)[reply]
The big bang is a theory supported by an overwhelming amount of evidence and almost univerally accepted by mainstream science. Some people choose to describe such things as "fact". Personally, I prefer to avoid the word. --Tango (talk) 00:25, 4 March 2011 (UTC)[reply]
But if I were to call it a fact, I wouldn't be wrong would I? ScienceApe (talk) 01:32, 4 March 2011 (UTC)[reply]
You might be, though so would the majority of cosmologists. Scientists prefer to use more cautious language when describing the current understanding of the universe. Theories are always being evaluated and refined. While I doubt that the Big Bang theory will be shown incorrect the way, say Phlogiston theory was, it's certain that the currently accepted cosmology will continue to undergo revision. Buddy431 (talk) 01:45, 4 March 2011 (UTC)[reply]
The Big Bang is the model that best explains a lot of evidence that we've seen (the evidence itself is definitely fact). Scientific theories aren't definite, final, and certain; they're all open to revision and replacement, and a lot of people don't understand or just don't like that, even though it's Science's greatest strength. In other words: good luck. -- Consumed Crustacean (talk) 01:52, 4 March 2011 (UTC)[reply]
(edit conflict with SA) The article History of the Big Bang theory is interesting as well, if a bit rough. Since the late 1960s, with the measurement of the Cosmic microwave background radiation, the Big Bang theory has become the overwhelmingly accepted model for the early history of the universe. In recent years, the CMB has been measured with greater and greater accuracy, with spacecraft such as the WMAP. While proponents of Non-standard cosmologies are not necessarily cranks, it is becoming increasingly difficult (both scientifically and socially) to entertain serious alternatives to the standard cosmology. Buddy431 (talk) 01:41, 4 March 2011 (UTC)[reply]
Well, there is a difference between a fact and a theory, but again it boils down to an argument of epistemology.. Take evolution for example, there is the FACT of evolution (that at its core) allele frequency varies over time in populations of animals; The THEORY of evolution proposes natural selection as the primary method by which this occurs. We have a Big Bang THEORY, but what is the "facts"? The facts are we have MBR and cosmic expansion and all those other things which perfectly fit the model of the big bang theory and no one has been able to come up with a better explanation, but will that ever lead to Big Bang being a "absolute fact"? I don't think it will, even if it is justified true belief. The problem as I see it is, generally "religious minded" people can't get it into their skulls that their doctrines which "claim" to be infallible and absolute are actually LESS reliable then science which makes no such claims and humbly accepts it doesn't know the absolute truth of much at all. Vespine (talk)

How about to get him to agree its a fact you say that his god was the one who made the big bang, because it was the only way to make other beings or whatever reason he is arguing. —Preceding unsigned comment added by 98.221.254.154 (talk) 03:18, 4 March 2011 (UTC)[reply]

You state that you'r debating with an "annoying religious guy". Such guys are not really after fact, or having their opinions up for reconsideration. It's a distinction between dialogue and rhetorics that's been debated since Gorgias. EverGreg (talk) 08:43, 4 March 2011 (UTC)[reply]

March 4

Refusal to investigate the existance of God.

Billions of dollars are spent to investigate the existance of the God Particle(Higgs boson) but scientists refuse to investigate the existance of God even though millions of people including the Pope,Cardinals,Imams and most polititions have constant contact with Him/Her and would be available to prove it.This would be of great value to world peace to prevent the religious divide facing everyone. John Cowell124.179.205.11 (talk) 00:12, 4 March 2011 (UTC)[reply]

The scientific method deliberately excludes religious faith. One cannot investigate the existence of something supported only by faith. Scientific investigation of the existence of God would be no different from investigating the existence of any deity, such as the Invisible Pink Unicorn, the Flying Spaghetti Monster, or any member of the Twelve Olympians in the Greek Pantheon. ~Amatulić (talk) 00:17, 4 March 2011 (UTC)[reply]
Pretty much every premise in the OP statement is wrong. Lots of scientists undertake investigation into the existence of God and you are free to undertake or fund any such activity you please. You might want to start with an article like Relationship between religion and science and go from there. Vespine (talk) 00:22, 4 March 2011 (UTC)[reply]
The OP should also note that "God Particle" is short for "Goddam Particle" because it is so elusive, but Lederman's publisher would not accept that name for his book. -- 119.31.121.88 (talk) 00:40, 4 March 2011 (UTC)[reply]
There have been several experiments to analyse the efficacy of prayer. Praying for someone else, who does not know they are being prayed for, shows no measurable effect. Due to the placebo effect, it is hard to check for an effect when the patient prays for their own recovery. For further reading, see this Google search for "double blind prayer study. CS Miller (talk) 06:36, 4 March 2011 (UTC)[reply]
Harvard did a study on cardiac patients, who when told they were being prayed for showed a statistically significant rate of worse outcomes, probably due to stress factors. Imagine Reason (talk) 19:38, 5 March 2011 (UTC)[reply]
If anyone really could prove the existance of god(s) to the same standard that is usually expected in science, then scientists would listen. The truth is, they can't. Faith and proof really don't go together. --Tango (talk) 00:23, 4 March 2011 (UTC)[reply]
I also disagree with Amatulic. There are many scientists that study this issue, amongst others Victor Stenger and Sam Harris argue that as long as religion makes claims about physical reality, there is no reason why science can not be brought to bear about the validity of those claims. You could argue that a pure definition of religion doesn't make claims about the "natural" world, only the spiritual or supernatural, but no religion fits this description. As soon as the spiritual or supernatural affects or interacts with our physical reality, which in all religions it does, then science can have a valid say. That's one of the reasons we don't like religious questions here, because they are very divisive and tend to degenerate into forum like discussions, which this site is not the place for. Vespine (talk) 00:30, 4 March 2011 (UTC)[reply]

Belief in Invisible Pink Unicorns does not cause the slaughter of innocent people.J.Cowell124.179.205.11 (talk) 00:46, 4 March 2011 (UTC)[reply]

Show me the Pope, Cardinal, or Imam who would be willing to submit any claims to measurable, scientific existence of God to scrutiny by disinterested, objective, and critical scientists. I have not seen them. Show me the claims they are making that they would allow external, unfettered investigation into. I have not seen them. I think you fundamentally misunderstand the situation if you think that religious leaders are interested in having their claims put to rigorous scientific scrutiny. They are not interested in cowing to scientific authority, much less risking the possibility of being wrong. (And even if you believe that God does exist, and does want to help you, you can still imagine a God that doesn't want his existence objectively proven and would hide it when push came to shove, because that would obviate the need for faith.) --Mr.98 (talk) 00:48, 4 March 2011 (UTC)[reply]
BTW it's not just faith that would be obviated (I like that word :). It's also free will. A provably omniscient God would not be compatible with free will. Free will requires a person to at least believe that their actions are their own (it's immaterial if the actions really are or aren't). Ariel. (talk) 01:15, 4 March 2011 (UTC)[reply]
While scientists can investigate (and disprove) concrete religious claims, like that the Earth is only a few thousand years old, you can't investigate the existence of God, since God can always "retreat" deeper. If we prove that nothing since the Big Bang requires the existence of God, you could then argue that God caused the BB. If we later prove what caused the BB, then God could be said to have created that, etc. StuRat (talk) 02:37, 4 March 2011 (UTC)[reply]
...essentially God of the gaps in a(n) historical rather than deeper-detail perspective. DMacks (talk) 04:45, 4 March 2011 (UTC)[reply]

The basis of science is to witness the universe in every form and derive the most accurate acount describing its modus operandi, and use this accumulated knowledge to make accurate deductions and predictions for that which cannot be observed.

Since God is not from within this universe, science cannot test for his exsistance or non-existance, there is no testable premise. All submitted evidense is subject to personal interpretation. Plasmic Physics (talk) 03:08, 4 March 2011 (UTC)[reply]

What I imagine the OP is saying is you'd go to someone who supposedly talks to God on a regular basis, like the Pope, and say, "so, Pope. Is there anything we could do that would prove the existence of God? Why don't you ask God about it and come up with something that we can test, e.g., look this direction on this day at this time and you'll see a comet the flies by and says, 'hi guys!'" I mean, if such a thing happened, and you could really be sure that it wasn't some kind of colossal hoax, even atheisty me would find it pretty interesting, maybe even compelling. But nothing like that has ever happened and nothing like that will happen. The Pope would never agree to such a thing. Whether that's because he knows it is probably outside of his ability to provide results, or because, again, those kinds of results would have severely problematic theological implications, or however you want to interpret it, it just isn't ever going to occur. No way, no how. It's not a case that scientists would ignore the Pope — I am sure the Dawkinsy types at the least would be happy to set up monitoring stations on the (highly likely) chance that the Pope would look like a fool in the process. But the Pope would not be foolish enough to partake in such a game, and would have perfectly good theological reasons for not participating. Feel free to insert any other religious figure for "Pope" in this paragraph. Every once in awhile such figures do hazard concrete beliefs about when a miracle will happen (the Earth will end next Thursday, etc.), but so far none of them have panned out. --Mr.98 (talk) 03:13, 4 March 2011 (UTC)[reply]

God wouldn't likely agree to it either, since it would defeat the whole faith aspect of God-based religions anyways. Plasmic Physics (talk) 03:21, 4 March 2011 (UTC)[reply]

That gives me an idea! Has anyone written a book or movie about a world where God DOES exist and is readily verifiable? Answers devout believer's prayers, punishes the wicked and performs regular miracles for example. Maybe investigating the contrasts between THAT kind of world and the one we live in would be an interesting exercise.. Vespine (talk) 03:42, 4 March 2011 (UTC)[reply]
There's a book where the protagonists can travel to all sorts of parallel worlds, and one of the worlds has an "active" God. The main difference is that no one can disbelieve, and that sins are punished instantly. It's not explored much else though. I think it's The Number of the Beast by Robert A. Heinlein, but I'm not certain. Ariel. (talk) 04:00, 4 March 2011 (UTC)[reply]
The Number of the Beast (novel) is a direct link to the page on Heinleins' book - 220.101 talk\Contribs 15:02, 4 March 2011 (UTC)[reply]

As far as I know, He only proved himself once before. In the Koran (Wikipedia excerpt), I've read that He lowered a table from the sky with a feast on it. Apparently, He wasn't very happy about having to do it, infact He sounded quite annoyed. Plasmic Physics (talk) 03:29, 4 March 2011 (UTC)[reply]

How does Plasmic physics know that God is not from within this universe? J.Cowell05:10, 4 March 2011 (UTC) —Preceding unsigned comment added by 124.179.205.11 (talk)

I don't know how PP knows it, but it seems to me an eternal entity could not be part of a universe (as we know it, according to science's current understanding) that has both a beginning and an end that does not allow for anything to exist outside of it. DMacks (talk) 05:30, 4 March 2011 (UTC)[reply]
Wow I so terribly do NOT want to start a debate but since when doesn't it allow for anything to exist "outside" of it? I've never heard that, for all we know, there might be an infinite amount of big bangs creating an infinite amount of universes completely separate to our own, in some other "dimensions" or something. Your view sounds "univercentric" to me ;) ... It might be impossible to know, but that's a log way from saying it is positively "excluded"? Vespine (talk) 05:54, 4 March 2011 (UTC)[reply]
The question specifically says "this universe", which I assumed meant the one we inhabit, rather than some other one or a superset/multiverse situation. DMacks (talk) 06:01, 4 March 2011 (UTC)[reply]

Our God-Bothering polititians will be most upset if you relegate God to another universe!.J.Cowell06:16, 4 March 2011 (UTC) —Preceding unsigned comment added by 124.179.205.11 (talk)

No, what I meant was, that He exists outside of everything, outside of time, space, reality. By "reality", I mean outside of our perspective. Think of it this way, all of the above is a very complex figment of His imagination. That's what I mean with outside. Plasmic Physics (talk) 06:33, 4 March 2011 (UTC)[reply]

Would you care to define in what way(s) that differs from non-existence as seen from an "in universe" POV? Roger (talk) 06:57, 4 March 2011 (UTC)[reply]

It is simply a matter of personal preference.

Are we getting of topic? Plasmic Physics (talk) 07:11, 4 March 2011 (UTC)[reply]

This whole thread is off topic for this desk and should have been boxed from the start. --Sean 14:30, 4 March 2011 (UTC)[reply]

If anyone is interested, in Contact_(novel), there is an excellent chapter where science's stand with respect to religion is explained. ManishEarthTalkStalk 15:12, 4 March 2011 (UTC)[reply]

Rather surprised that in this supposedly enlightened age there are still those who wish to shut down discussions on the most fundamental of all scientific questions.Shades of Galileo,Scopes and book burning.J.Cowell124.185.229.128 (talk) 00:01, 5 March 2011 (UTC)[reply]

No, you miss the point. It is not a scientific question at all. Individual scientists no doubt have their own beliefs one way or the other about God, but science per se is not concerned with the matter. It is a question of personal faith, period. Nobody has ever proven that God exists, or that God doesn't exist - in a way that would satisfy science - and they never will. That's why it's not a relevant topic for the Science reference desk. -- Jack of Oz [your turn] 00:11, 5 March 2011 (UTC)[reply]
Im confused. Why does it matter if God exists or not? what will it change? If he does exist then we still wont be able to explain anything, we will still be trying just as hard to figure all this stuff out. Its not like if we prove he exists, he will come out from behind the clouds and say ok you got me here are the answers to all the questions you could ever ask. We will be in the same reality either way. —Preceding unsigned comment added by 98.221.254.154 (talk) 01:18, 5 March 2011 (UTC)[reply]

Flower identification

Sitting on a coworker's desk in NE China. The plant is basically a 2 dimensional green hand shape. This flower appeared at the very tip of one of the "fingers". Can anybody tell me what plant this is? purple and white flower It's quite fragrant. The Masked Booby (talk) 01:44, 4 March 2011 (UTC)[reply]

Looks to me like Neomarica northiana, part of the Neomarica (Walking Iris) genus. Nanonic (talk) 02:04, 4 March 2011 (UTC)[reply]
I believe you've nailed it! Thank you. The Masked Booby (talk) 02:08, 4 March 2011 (UTC)[reply]
That's a beautiful picture too. If it's yours, would you upload it to Commons, now that you know the name of the plant? ~Amatulić (talk) 21:17, 4 March 2011 (UTC)[reply]

Einstein in the Manhattan Project

Why didn't Einstein get clearance for working in the Manhattan Project? It is true that he was left-leaning, but being the genius that he was, and the Einstein–Szilárd_letter, shouldn't anyone consider him more of an asset than a risk? Quest09 (talk) 01:48, 4 March 2011 (UTC)[reply]

By that time physics had moved on well beyond Einstein, who was more of a figurehead than a leading physicist. StuRat (talk) 02:29, 4 March 2011 (UTC)[reply]
They approached Einstein very early on with the possibility of getting help from him on the theory of gaseous diffusion. They found him to be kind of useless — the sort of work Einstein did well was fundamental and theoretical. The kind of work that was really required to make an atomic bomb was specific and literal and practical. It was in many respects more of an engineering task than a scientific one, albeit it was an engineering task so bleeding-edge that it required huge teams of scientists to work out the details. He was considered a security liability because of his strong political beliefs, as well. But mostly the people who ran the project couldn't see much use for him. Building a bomb required nuclear physicists, engineers, chemists, metallurgists, etc. It didn't require people who had clever ideas about the structure of space and time. It was really not Einstein's forte.
By contrast, Niels Bohr they did bring into the project, even though he also didn't do much of practical value. (He helped with the theory of the neutron initiator, but that's the only practical problem he was involved in, as far as anyone knows.) Bohr was more of a morale builder than anything else. It's also not the case that Einstein was really "beloved" by that generation of physicist in the way that Bohr was; it's not clear to me he would have helped morale. Einstein was also seen by the quantum theorists as kind of backwards for his long-standing debate with Bohr. He also had essentially no students, so it's not like people looked up to him fondly for that reason, the way that many did with Bohr. --Mr.98 (talk) 02:58, 4 March 2011 (UTC)[reply]
(ec) Some searching in Google Books turned up The Einstein File by Fred Jerome, which appears to be well researched and cites primary sources. Chapter 4, "Banned from the Bomb", is all about this question. The short version is that there's not enough evidence to be sure, but the author thinks that it's J. Edgar Hoover's fault. You have to remember that this is a bureaucracy—Roosevelt trusted Einstein, but it wasn't Roosevelt's job to make the decision. Incidentally, Einstein did not request clearance; Roosevelt requested it for 31 scientists including Einstein. I somehow doubt that Einstein would have agreed to work on the Manhattan Project in any case. What StuRat and Mr.98 said is also true (Jerome mentions this as well). -- BenRG (talk) 03:12, 4 March 2011 (UTC)[reply]
I'm not sure that FDR really trusted Einstein on that level of things. It's one thing to trust him when he says, "Germany's trying to get us, look into this," and even then, FDR didn't really put a lot of money behind that. That's not the same thing as saying, "please run my bomb project."
If Einstein had really been valuable to the project, they probably could have decided to let him in. They made a lot of security exceptions during the war — Oppenheimer was super lefty, practically Communistic, at points in the 1930s, as all well knew. Arthur Compton was a member of various "fellow traveler" organizations. There are other examples as well. Because these people were actually quite essential to the work, Vannevar Bush and later General Groves were willing to look the other way, so long as close tabs were kept on them. With Einstein, it wasn't worth the risk.
It's worth also qualifying what is meant by "risk" here — it isn't necessarily that he's a spy or a saboteur. It could also just be that he's not good at keeping a secret, which is a less vicious, but equally damning, concern.
It's of note that the period in which Vannevar Bush thought about trying to get in touch with Einstein on this, the Manhattan Project was still in the extremely exploratory stages. Maybe at that stage having someone like Einstein would have been useful. But once you get into 1942 or so, he would not have been useful at all. --Mr.98 (talk) 03:23, 4 March 2011 (UTC)[reply]
Einstein made his "practical technologist" bones early on in his career, back in the era when Edison and Tesla were inventing useful things. Didn't he work in a relative's electric motor factory? Didn't he invent some type of refrigerator? And finally, didn't he review myriad patents? He still had the same brain, despite the decades of theoretical physics, and he could have been of great use to the project in many practical areas, let alone theoretical physics. Edison (talk) 04:51, 4 March 2011 (UTC)[reply]
Sure, very early on he did that sort of thing, with a previous era's technology. But the guy was practically retired by the time of World War II. He spent his time hanging out on the beach and having coffee with Gödel and fruitlessly trying to come up with the Unified Field Theory. He was out of the game. (Oppenheimer certainly felt that he was, in any case. There is some snarky quote that Oppenheimer gave the magazines just after the war about how Einstein was a signpost of where physics had been, not a sign pointing to where it was going.) You really imagine him going to Los Alamos and breaking a sweat on how to design a diffusion plant, or going to Chicago and engineering the first reactors? If he had been some kind of great organizer, the type of guy who could run a big project, I'd agree with you. But he never really displayed a talent (or interest) for that kind of thing. In his work, the stuff he was great at was coming up with the big, grand theory. He was not great at doing millions of calculations, much less quickly. The kind of work that was required to build the bomb was millions of calculations, not big, grand theories. A few clever ideas, sure. But they had other idea people, once the program really got under way. --Mr.98 (talk) 12:43, 4 March 2011 (UTC)[reply]
Were it not for security worries, it would have made sense to use the far from ancient and doddering mind of Einstein in the defeat of the Axis, if he were willing to help. Einstein was only 63 in 1942. Richard Feynman was 68 when he played an important role in the Challenger shuttle disaster investigation, which was in an area of technology he had zero background in. Even if Einstein was squeamish about building bombs to drop on Hitler, there were many areas of war research he could have helped with, just as Thomas Edison worked on acoustic detection antisubmarine apparatus and synthetic phenol in WW1, 1914-1918, from age 67 to 71. At a mundane level, Einstein could have screened patent applications to pick out those which actually were physically practical and which could have use in war equipment or in any area useful to the war effort such as communication, electricity, transportation, food preservation, or refrigeration. Edison (talk) 16:30, 4 March 2011 (UTC)[reply]
Well, to me the real question is to ask whether he could have worked on other projects, and why he wasn't used more readily there. There were plenty of projects of far lesser security concern than the bomb — things that were graded only "secret" or "confidential" rather than "top secret." An indeed, Einstein did do some consulting with the Navy. But I just don't seem him packing up and going to a government lab, myself. (And, entirely separately, I would argue that Feynman's background — including his war work! — actually made him much more qualified for a Challenger-like investigation than Einstein would have been. Feynman had lots and lots of practical experience in debugging large systems; that's actually what he did during his work with Oak Ridge during the war on radiation safety, on the possibility of critical masses forming inadvertently, if I recall. I don't see any similar experience on Einstein's resume. Einstein and Feynman were both great theoretical physicists, but they had very different minds. Feynman's strikes me as the far more practically oriented.)
Incidentally, there was, I do recall, a physicist who screened said patent applications for the Office of Scientific Research and Development in the manner that you describe (but we lack an article on him). It probably would have been a job Einstein could have, in fact, done quite well if he had been asked to, and wanted to. --Mr.98 (talk) 16:51, 4 March 2011 (UTC)[reply]
Physicists at MIT dropped everything to work on radar(secretly). By contrast, it would have been a morale builder if some aging science superstars such as Einstein were shown in Life magazine in a lab somewhere "doing war research" even if it was not really the forefront. A mathematician or physicist is typically far over the hill in terms of major breakthroughs by his early 60's but they are the bedrock of review panels, with experience and perspective to decide what is promising, and what is a blind alley that has been gone down before. I knew women who drove over an hour each way to work a long shift building artillery shells and airplanes in WW2, and it seems unconscionable to have brilliant physicists "sit the war out" rather than making whatever contribution they were capable of to keep Hitler and Hirohito from dividing up the spoils after defeating the Allies. Edison (talk) 16:59, 4 March 2011 (UTC)[reply]
Mm, perhaps, but we're getting a bit far afield, now. We've gone from discussing whether Einstein would have been useful for the Manhattan Project, to whether he would have been useful as a bit of propaganda. If I were a government scientist-administrator like Vannevar Bush (who was on the cover of Time magazine for his war research), I wouldn't necessarily want to give Einstein such a soapbox. Einstein's political views were fairly out of the mainstream for the period (even if most of them are pretty tame by modern standards), and propping up someone as a centerpiece can backfire on you. (Would Einstein have followed the "party line" if it went against his convictions? I'm doubtful.) Anyway, I'm not saying that the past had to happen the way it did, but I do think their decisions were defensible along both practical and scientific grounds. Personally I think Einstein is pretty out of step from the kind of scientist who was really quite useful in World War II. He's not a Bush, Oppenheimer, Compton, Szilard, or a Teller. This in no way should be seen as disparaging of Einstein as a scientist, or as a person. --Mr.98 (talk) 20:24, 4 March 2011 (UTC)[reply]
There are women (and men) right now who drive more than an hour each way to work long shifts building artillery shells and airplanes. I don't see how this is relevant. --140.180.5.6 (talk) 17:29, 4 March 2011 (UTC)[reply]
Sorry for your lack of comprehension of what I was trying to say. You may be unfamiliar with the fact that people (our moms and grandmothers) abandoned their typical lives during WW2 to devote their full time to the war effort, travelling long distances (probably taking far longer than an hour in many cases) at the wartime 45 mph speedlimit on bald tires to do hazardous unfamiliar work. Einstein was basically left sitting on his butt, maybe scribbling unified field equations that could not work on a blackboard,and did not aid the war effort, if statements above are correct. He might have been able to do more to win the war than "Rosie," my ancestor who worked double shifts making antiaircraft rounds, and once setting one off by accident in the crimper, by working faster than the machine was set up for, while Albert sat on his butt, unused and ignored. A wasted Einstein seems criminal. Maybe he could have worked some double shifts and devised a proximity fuse for artillery shells which was less prone to explode over friendly troops on the way up when it should have exploded over enemy troops on the way down, as Audie Murphy reported in his war memoirs, or maybe he could have devised a magnetometer an airplane could have used to locate enemy subs, as the Japanese devised toward the end of the war, or night vision goggles as the Germans devised toward the end of the war, and saved thousands of Allied lives and shortened the war. Edison (talk) 05:11, 5 March 2011 (UTC)[reply]
In my web searches the one thing which people are most eager to credit Einstein for in the second World War is the "Philadelphia Experiment". Now I don't believe the crazy stories about that event, but I think it was typical of the military at that time to try to drown out accurate information with wild disinformation (such as covering up experimental aircraft or surveillance balloons with flying saucer alien stories). I reserve suspicion that even after all this time, there might still be some tech he was really working on, perhaps stealth technology for ships - something basic, e.g. "antisound" for radar, hmmm, or for subs... - which remains sensitive enough that Einstein still hasn't been given his due, except by fringe sources. Wnt (talk) 17:33, 4 March 2011 (UTC)[reply]
The problem is that there is literally zero reason to suspect this, other than "it'd be interesting if it were true." --Mr.98 (talk) 20:27, 4 March 2011 (UTC)[reply]
Oh, you're absolutely right. But there are many things from that war that remain absolutely secret, and thus many histories that seem a little implausible. Like Glenn T. Seaborg's discovery of so many elements after the war, as opposed to, hypothetically, while trying to build a better neutron initiator. We have to indulge our imaginations now and then in order to avoid believing systematic error is fact. Wnt (talk) 00:23, 5 March 2011 (UTC)[reply]
I'm not sure I gather your Seaborg analogy at all — Seaborg discovered all those elements because he had a total monopoly on the tools necessary to discover said elements. There's no real mystery there. It's also not like he wasn't working on the weapons program at the same time — he was, in various capacities. While it's always possible there are secret programs out there that have not yet been discovered, there is really no reason to assume any science fiction. It's also hard to imagine that programs of the scale of what you're describing would have existed with absolutely no evidence having surfaced thus far. --Mr.98 (talk) 02:24, 5 March 2011 (UTC)[reply]

Time-travel photography

Suppose one has an opportunity to travel to the past and take photos in, say, ancient Greece or Rome (assuming that his/her activity wouldn't affect the subsequent events). Is there any way to prove that the photos are genuine upon return to our times, making such activity worthy? —Preceding unsigned comment added by 89.76.224.253 (talk) 12:24, 4 March 2011 (UTC)[reply]

You could test for evidense of image manipulation eg, CGI Plasmic Physics (talk) 12:37, 4 March 2011 (UTC)[reply]
There are ancient structures (such as remains of houses) that have been found and identified, but not yet excavated. You could bring a list of such structures and scratch "89.76.224.253 WAS HERE!" on a few of them (to make sure at least one inscription survives) and take pictures. Sjö (talk) 14:11, 4 March 2011 (UTC)[reply]
They don't have to be structures that have yet been found—collecting photographs which successfully and accurately guide new present-day archaeological exploration would be an extremely potent evidence. If our hypothetical scenario allows the photographer to leave physical evidence and objects behind, the science-fiction-story-approved method is to bury a small artefact containing a known quantity of one or more radioisotopes in a suitably sealed, tamper-resistant, temper-evident container. TenOfAllTrades(talk) 14:36, 4 March 2011 (UTC)[reply]
BTW. Even if time-travel is impossible, can information be transmitted from one time to another? 212.169.188.228 (talk) 14:48, 4 March 2011 (UTC)[reply]
Sure, but only in one direction — past to future. Transmitting information from the future to the past is time-travel, just dressed up in different lingo ("information" must be transmitted by a carrier, and the carrier is a "thing" you are sending back in time). --Mr.98 (talk) 14:58, 4 March 2011 (UTC)[reply]
I'm not sure why it's so frequently stated on the science ref desk that backwards time travel is impossible. That's not what it says in the article. In answer to the question, I suspect the kind of physicists who think about time travel would make little distinction between information and matter, and tend to regard one as equivalent to the other, but IANAP. 213.122.8.112 (talk) 15:15, 4 March 2011 (UTC)[reply]
Photographing the stars will set the time period when you are talking about going back hundreds of years. Of course, any photo of anything is easy to fake now - and will surely be easier to fake in the distant future when time travel might become possible. -- kainaw 14:49, 4 March 2011 (UTC)[reply]
If some laaarge planet was made of, say, highly shiny metal, and you had an amazing telescope, you could use the reflection to photograph Earth (and possibly its inhabitants) a few million years ago. Finding such planets that show you the past ten thousand years are a bit hard though... Anyways, as of now, we can't even see planets, much less view your reflection on them.

Information can only be transferred forward in time. There are two ways of transferring information, one is with materials (a gold plaque, light, radio waves, paper, etc), and another is with quantum mechanics. Materials can't be sent back in time (because time machines destroy themselves 10-43 seconds after they are created), and with quantum mechanics, though it can send data faster than light, you can't create a Tachyonic antitelephone with it, for various reasons. If you want some links for more information on sending information into the past, see: Tachyonic antitelephone, and Causality (physics). ManishEarthTalkStalk 15:06, 4 March 2011 (UTC)[reply]

There are a number of fiction stories with plots very similar to this. Hero gets some sort of past viewer, takes some photographs, then can't sell them because his photographs contradict in some ways with modern archeological thought. Therefore everyone assumes they're expertly made, but poorly researched fakes.
I think your best bet would be to use your past-viewer to identify some archeological sites that have not yet been discovered, then very publicly predict where they will be. (So that, after the fact, there can be no doubt that you were the first to suggest their existence.) APL (talk) 15:54, 4 March 2011 (UTC)[reply]
... and here's one of me with a dodo. Tasted a bit like chicken ... Gandalf61 (talk) 16:37, 4 March 2011 (UTC)[reply]
Two ideas: (a) Photograph a familiar large natural object (I'm thinking about a mountain) that existed both then and now, and which has been changed by human development, preferably a thousand years ago, perhaps via a strip mine, something like mountaintop removal mining, or a lot of housing construction; (b) Take pictures of the night sky through a telescope (no light pollution!) and when you come back, have an astronomer friend verify that the different star and planet positions align with the date that you were visiting. For bonus points, take a look at list of novae before you go, and make sure to photograph areas of the sky that contain those stars. Neither of these ideas will provide irrefutable evidence that you did travel back in time, but you may be able to convince some Experts, on the grounds that a layman would not really be able to simulate those particular photos. While you're at it please bring back a photo of The Sphinx with its nose; I remain curious. Comet Tuttle (talk) 18:43, 4 March 2011 (UTC)[reply]
Naturally, but I presume that proving the portraits of people is impossible... —Preceding unsigned comment added by 89.76.224.253 (talk) 21:48, 4 March 2011 (UTC)[reply]
Would that be the sphinx in ancient Greece or Rome? Cuddlyable3 (talk) 05:43, 6 March 2011 (UTC)[reply]

Deepwater Horizon oil spill: Why the glacially slow testing of the blowout preventer?

The Deepwater Horizon oil spill article says that the blowout preventer, which failed to seal the pipe, was removed for analysis on September 4 2010, that the investigation by NASA began November 16, 2010, and that it was still ongoing December 23, when the U.S. Chemical Safety Board called for for the investigation to be taken over by others due to a perceived conflict of interest. NASA now been investigating this device (which has no apparent connection to aeronautics or outer space) for 3 1/2 months, with no expected date announced for a final report on how it malfunctioned, other than occasional "leaks" about a dead battery and leaky hydraulics. There is nothing like a deadline to sharpen the focus of an investigative team. When are the investigators expected to issue a report on the blowout preventer? The most recent news stories I found, from January 2011 said a "final report" on the oil spill was issued January 11, without the report on the preventer, which was expected in February 2011. I have seen no press coverage of the preventer analysis final report.Is this one of those cases where they wait until the press has lost interest, or has the government progressed from data collection and analysis to spin doctoring of the language in the findings? Edison (talk) 15:59, 4 March 2011 (UTC)[reply]

Here are some possible questions to ask as to why it hasn't already been done.
Should NASA drop everything else and divert their limited resources to this one issue?
Should NASA disrupt its current timetable, (which will have been planned sometime back and have other areas, departments, experts, etc., already committed to the current times and places)?
Should NASA just get on and pull apart the blow-out preventer before they have had time to come up with a comprehensive plan to avoid destroying any evidence due to oversights?
Should NASA just place the blow-out preventer in any of their unused lots and pull it apart there, or should they schedule it to be tested in a suitable or custom built test bed (pressurised tank or what ever) which might enable a better scientific analysis to be carried out -so that with the knowledge gained, the occurrence of future oil-spills at sea can be reduced? --Aspro (talk) 17:04, 4 March 2011 (UTC)[reply]
This is just me, but I think the OP was more interested in his other questions: namely, why NASA is investigating this in the first place, even though it has nothing to do with aerospace. --140.180.5.6 (talk) 17:16, 4 March 2011 (UTC)[reply]
I have over stated NASA's part in this. It hasn't been sent to NASA for their aerospace know-how, rather their project oversight skills and their ability to provided the basic facilities, site security, etc., were the work can be carried out by whoever is brought in to do it. NASA isn't doing the work themselves no more than they manufature rockets and things--Aspro (talk) 19:56, 4 March 2011 (UTC)[reply]
From your comments, it just sounds like NASA is Way Too Busy to do this crucial analysis, which bears on possible dangers lurking in countless other blowout preventers, which might be similarly unable to perform their only known function. Maybe the analysis should have been given to some university, or some other group with no knowledge of petroleum extraction, but with "project oversight skills and their ability to provided the basic facilities, site security, etc." like maybe the Federal Reserve, the FBI, the National Security Agency or the National Bureau of Standards. Edison (talk) 05:00, 5 March 2011 (UTC)[reply]
Here a news report on it: [3]:

'"The condition of the BOP, as well as additional unanticipated events, were the reasons for the delay," the joint investigation said in a statement.'

Somebody should have asked what the "unanticipated events" were. Perhaps the press has already lost interest. Some appear to suspect sabotage of the investigation:

'A lawmaker and the Chemical Safety Board have questioned whether the integrity of the BOP testing has been damaged by the involvement of a Transocean employee who served as the subsea supervisor on the rig that was drilling the Macondo well.'

StuRat (talk) 21:57, 4 March 2011 (UTC)[reply]
Thanks, Stu, that news report is exactly what I was looking for. Strange how much the phrase "unanticipated events" sounds like someone tap dancing while they whistle "Tea for Two" as they try to think up phrasing to make unacceptable blunders seem like acts of God. Edison (talk) 05:40, 5 March 2011 (UTC)[reply]
Edison, you may also be interested in reading the National Academy of Engineering's special committee report, commissioned by the President: Analysis of Causes of the Deepwater Horizon Explosion, Fire, and Oil Spill to Identify Measures to Prevent Similar Accidents in the Future. Interim Report on Causes of the Deepwater Horizon Oil Rig Blowout and Ways to Prevent Such Events is available for free to the public. This was a preliminary investigation; the report was published in November; in addition to preliminary technical analysis, one of the critical items they determined was that a suitable agency does not currently exist to evaluate deepwater drilling safety or to investigate accidents. Several preliminary policy-level suggestions were made. In fact, it was this November report that preceded all of the events (such as NASA involvement)that you are currently expressing frustration with! One of the reasons NASA has been doing technical analysis was its role as an uninvolved Government agency; it was pointed out that the Mineral Management Service, who oversaw deepwater drilling, had serious conflict of interest because the same agency collected revenue and enforced policy (a unique case in the Government - compare the IRS and, say, the FBI). MMS has since seen a major organizational overhaul, by presidential decree; NASA was an "independent" agency who would play the role of unbiased Government advocate; and minimize the risk of any crony-ism or direct conflict of interest. But this is also a reason why technical analysis has been going so slow. The engineering analysis of the accident is confounded by legal-ese, decision-by-committee, and government regulation. I had the (mis)fortune to hear a presentation by the NAE committee right before they released their November report - and what I gathered was that the scientific and technical results must all be revealed very strategically to avoid poisoning any legal process that is currently taking place. (And there is some heavy, multi-billion-dollar litigation taking place amongst all involved parties). Nimur (talk) 13:13, 5 March 2011 (UTC)[reply]

Where do very sticky stickers come from?

Someone put up an offensive picture on a utility pole near my home and it can't be peeled off. What to do besides calling the city? 12.199.96.253 (talk) 16:18, 4 March 2011 (UTC)[reply]

Is this a "science" question? Bus stop (talk) 16:20, 4 March 2011 (UTC)[reply]
At your local hardware store, ask them about solvents that remove stickers. I've a little bottle of very nasty stuff that dissolves basically anything sticky within a few seconds. Not very expensive. Very handy thing to have around in life more generally. I'm sure the stuff is toxic as all get-up, but every once in awhile you actually like to get things clean... --Mr.98 (talk) 16:33, 4 March 2011 (UTC)[reply]
Acetone might work. Not good to inhale, or otherwise ingest. Bus stop (talk) 16:37, 4 March 2011 (UTC)[reply]
Acetone's pretty darn safe to both inhale and ingest: [4]. Buddy431 (talk) 23:41, 4 March 2011 (UTC)[reply]
Look for a liquid called "Goo-Gone" at the hardware store (The dam spam filter prevented a link to a useful article at E-how). It is useful solvent for removing adhesive stickers. You might have better luck with the utility company than the city, since the city does not own the pole, although if the Mayor complained to the utility it might motivate them. Edison (talk) 16:47, 4 March 2011 (UTC)[reply]
Would it work on stickers that don't come off? YouTube videos primarily show adhesives being removed, but what if the sticker itself doesn't come off? Imagine Reason (talk) 22:33, 4 March 2011 (UTC)[reply]
Just because someone else has interfered with a utility pole, does that remove the duty of another citizen to report you to the authority's for committing further possible damage simply because you can't stop your sensitivities from being activated. Should we be encouraging individuals to take things into their own hands like this by answering such a question? Say the bottle of solvent broke and it ignited, other people could be left without a utility until its fixed. --Aspro (talk) 17:18, 4 March 2011 (UTC)[reply]
I'm pretty suspicious of the idea that you could really interfere with the utility pole with some solvent. We're not talking about bathing the pole in gasoline. Topical application. And yes, I think in today's day and age, encouraging people to take a little responsibility for their communities is not a bad thing, assuming it is not a really dangerous or hazardous recommendation. I certainly wouldn't expect MY city council or utilities company to do anything about it, with their track record. If citizens do not take some responsibility for public spaces it quickly becomes a tragedy of the commons situation. --Mr.98 (talk) 18:19, 4 March 2011 (UTC)[reply]
I suppose a black felt tip pen is out of the question. Richard Avery (talk) 19:34, 4 March 2011 (UTC)[reply]
For some reason my various permanent markers never seem to go really dark when I really need them to. Imagine Reason (talk) 20:53, 4 March 2011 (UTC)[reply]
Some other things that might work:
WD-40. Still smelly and messy, but not quite as toxic as acetone. Less likely to damage the paint (if any) on the pole than acetone.
Actually I believe WD-40 is more toxic than acetone. Acetone is not very toxic at all. Ariel. (talk) 22:24, 4 March 2011 (UTC)[reply]
Yeah, you can literally drink acetone (not without ill effects, but you aren't likely to die or have any long term impairment). One guy drank 200 mL (about a cup) and turned out fine (eventually) [5].
Peppermint oil (or clove oil, etc.). Smells pleasant and less toxic (although I don't suggest you drink a bottle). Can damage plastics, and might damage some paints. StuRat (talk) 21:30, 4 March 2011 (UTC)[reply]

Goo Gone and a scraper (such as a putty knife) should do the job.--Srleffler (talk) 18:16, 5 March 2011 (UTC)[reply]

Marshmallows in the microwave

What causes marshmallows to expand in size when microwaved? And why do they collapse when you take them out? Googlemeister (talk) 17:11, 4 March 2011 (UTC)[reply]

At a guess, I would say that as their made by having air whipped into them, then it is this air that is expanding!--Aspro (talk) 17:20, 4 March 2011 (UTC)[reply]
The air expands when it is heated by the warming marshmallow, the marshmallow cools when it comes out of the microwave and cools the air which shrinks. That combined with the very elastic substance of the marshmallow which allows the air to expand but retains it encapsulated inside the goo.Richard Avery (talk) 19:30, 4 March 2011 (UTC)[reply]
Air doesn't expand much when heated, but water expands dramatically when boiled. Since microwave ovens use a frequency designed to preferentially heat water, it may boil while the rest of the marshmallow is still well below the boiling temperature of water. Turning off the oven would allow the water vapor molecules to quickly cool and condense back into liquid water. StuRat (talk) 22:00, 4 March 2011 (UTC)[reply]
Ditto. (edit conflict) Plasmic Physics (talk) 22:06, 4 March 2011 (UTC)[reply]
Note that it isn't really true microwave ovens use a frequency designed to preferentially heat water. See Microwave oven#Principles. It's just that Dielectric heating works better on water then other substances commonly contained in food. According to the last post here [6], nitrobenzene and chloroform can also be efficiently heated, but most of us don't want either in our food... Nil Einne (talk) 22:44, 4 March 2011 (UTC)[reply]

An infestation of ladybugs

Okay, I read Coccinellidae#Infestations. It doesn't really explain what I'm seeing in my condo unit.

Nearly every day for the past year, I find a ladybug in our home, buzzing around our dining room chandelier at dinnertime or just crawling up a wall or window. I dutifully catch it and throw it outside. There certainly was never a "swarm" that flew in, as described in the article. We see just one at a time. We'd notice a swarm because they'd have no place to hide, no air ducts, no plants. I suppose the only inaccessible spaces in our home are the spaces behind the kick panels under our kitchen and bathroom cabinets, but I see no reason why a ladybug would want to go into such dry dusty places.

There are plenty of other bugs outside (flies, mosquitoes, moths, etc.), far more numerous than ladybugs, that could fly in or crawl in when we open the door. But we don't see those, except for an occasional crawler like a spider or silverfish. We just see a ladybug about once a day.

I'm skeptical that every day one ladybug waits outside to seize an opportunity to fly into our door on the brief occasions when it opens. And I'm pretty sure they aren't hitch-hiking in on our clothing. And I'm quite certain that I'm not throwing out the same ladybug every day!

I have wondered if they're breeding in our house, but I've never seen any larvae, and I don't know how they'd survive in our home with no plants and no aphids — unless they're eating dust mites or other critters too small for my vision.

Any thoughts about what might be going on? I don't mind the ladybugs, but I'm curious to know how they get in, when other more numerous bugs don't. ~Amatulić (talk) 21:11, 4 March 2011 (UTC)[reply]

How can you be certain they aren't on your clothing when you come home ? Do you fully check yourself in a mirror to be sure you don't have one on your back ? StuRat (talk) 21:17, 4 March 2011 (UTC)[reply]
These are persistent creatures, capable of finding small crevices, and they show up in the oddest places - between panes of double pane windows, for example. I have little doubt that they'd be coming in through some crack, but finding it may not be easy! Many references like [7] can be found using "asian ladybug cracks" for search terms. Wnt (talk) 00:16, 5 March 2011 (UTC)[reply]
Thanks for the responses.
StuRat: Actually, yes. I take my jacket off and check it. Even if they do come in on my clothing, I seriously doubt that one hitches a ride every day. It isn't the time of year for seeing ladybugs outside either.
Wnt: They may be coming through cracks, but as I said there are other more numerous crawly things around that can fit through cracks also, but don't. And why they seem to trickle in at a rate of one a day mystifies me. I think seeing them between panes of a double pane window may be the result of a larva crawling through a crack in the seal of the window pane and then metamorphosing between the panes, since those are rather flat soft creatures while the mature beetle is round and hard and unlikely to fit. ~Amatulić (talk) 01:22, 5 March 2011 (UTC)[reply]
Hello. I manage a hotel, and your very same question was a mystery to me for years. There would alsways be one or two lady bugs randomly crawling around somewhere in our lobby. In my experience to your question, there is, somewhere in your home, an infestatation of lady bugs...you just haven't found it yet. Ours was way up in the outside corner of an air duct in the ceiling. Not inside the duct, but rather in between the metal tubing and the wooden housing. Needless to say, they don't really hurt anything, so aside from fogging your house with professional grade chemicals, a lady bug or two crawling around is not so bad relatively speaking. Maybe you should get a houseplant and give them somewhere to hang out ;) Cheers! -- David Able 04:37, 5 March 2011 (UTC)[reply]

Pouring liquids

I'm not sure whether the question was already answered earlier, but anyway. When I start to pour juice for example, the pack shortly afterwards starts to "breathe" and "exhales" some quantities of juice, disrupting the normal flow. Why is that and how to avoid it? —Preceding unsigned comment added by 89.76.224.253 (talk) 21:58, 4 March 2011 (UTC)[reply]

It's because you are creating a partial vacuum in the container by removing part of the contents without replacing it. An air hole in the opposite side is a simple cure, allowing air to replace the missing contents. If that's not practical, pour slowly enough that the opening is not entirely blocked with liquid, and air will get sucked back in that way.
If you don't do either, the partial vacuum will build until it sucks the some juice and air back in, possibly going slightly over atmospheric pressure, and will then "spit some back out", to equalize the pressure. StuRat (talk) 22:02, 4 March 2011 (UTC)[reply]
To expand on that, making an air hole is very easy. Just get something sharp (knife, scissors, pin, whatever) and make a small hole in the top of the carton away from the opening. You'll find even a very small hole makes a really big difference. --Tango (talk) 23:22, 4 March 2011 (UTC)[reply]
Use a Churchkey, a device specifically designed for the job (with cans, anyway). Buddy431 (talk) 18:51, 5 March 2011 (UTC)[reply]

Apple pH

What would be the approximate pH of a common-or-garden eating apple? DuncanHill (talk) 23:35, 4 March 2011 (UTC)[reply]

Probably about the same as the juice, on the acidic side between 3.5 and 4.0. Not as acidic as citrus. There's a list of pH levels from the USDA for various fruit juices here. ~Amatulić (talk) 01:28, 5 March 2011 (UTC)[reply]
Thank you. DuncanHill (talk) 03:48, 5 March 2011 (UTC)[reply]

March 5

An attempted desription of Eternity,

Would someone identify the writer of this attempt to illustrate the idea of Eternity which I read decades ago and will now try to re-create. Try to imagine a block of granite 100 miles long by 100 miles wide by 100 miles high.Once every 10,000 years a small bird alights on it to sharpen it's beak.After the last bird has worn away the last granule of the block this will represent the first second of Eternity. J.Cowell00:27, 5 March 2011 (UTC) —Preceding unsigned comment added by 124.185.229.128 (talk)

I think it may be HG Wells in his book about the history of the world. 92.24.182.238 (talk) 00:44, 5 March 2011 (UTC)[reply]
It sounds like a bit of the sermon on Hell in A Portrait of the Artist as a Young Man by James Joyce. AndrewWTaylor (talk) 13:34, 5 March 2011 (UTC)[reply]
Yes, it is from Portrait:

Now imagine a mountain of that sand, a million miles high, reaching from the earth to the farthest heavens, and a million miles broad, extending to remotest space, and a million miles in thickness; and imagine such an enormous mass of countless particles of sand multiplied as often as there are leaves in the forest, drops of water in the mighty ocean, feathers on birds, scales on fish, hairs on animals, atoms in the vast expanse of the air: and imagine that at the end of every million years a little bird came to that mountain and carried away in its beak a tiny grain of that sand. How many millions upon millions of centuries would pass before that bird had carried away even a square foot of that mountain, how many eons upon eons of ages before it had carried away all? Yet at the end of that immense stretch of time not even one instant of eternity could be said to have ended. At the end of all those billions and trillions of years eternity would have scarcely begun.

. See here for the full text. AndrewWTaylor (talk) 13:38, 5 March 2011 (UTC)[reply]
I'm not convinced that is it. I still recall something in one of the several history books by HG Wells. 92.15.18.16 (talk) 14:41, 6 March 2011 (UTC)[reply]

What makes tape sticky

I find myself on this board, randomly contributing to answer a question, and figured I'd throw one out there that I came across today. So, I was doing my expense report and taping receipts to a piece of paper for photocopying, and I wondered: What makes tape sticky? I mean, I know it has something to do with an adhesive, and tacki-ness (sp), but more specifically what exactly is going on? Is it a chemical bond? What physical process is going on that makes it stick, but not bind permanently? Also, it seems when you peel scotch tape off paper, the tape is "less" sticky but the paper is not sticky at all. Is it absorbed? Deep thoughts, by David Able 04:51, 5 March 2011 (UTC)[reply]

Adhesion is the article, but it's unfortunately confusing. The most important thing is that adhesives flow into microscopic gaps in the surface, giving a much larger contact area. In the case of scotch tape that happens when you press the tape against the surface. When you pull something apart (i.e., put it under enough tension that it breaks), the weakest bonds break first. If you're lucky those will be the bonds that formed when you applied the adhesive, but that's not a foregone conclusion. No bond is really permanent. As for the actual bonding mechanism of scotch tape, I'm not really sure. It's some kind of van der Waals force. -- BenRG (talk) 11:07, 5 March 2011 (UTC)[reply]
I keep wondering if electrostatic attraction isn't involved in "stickyness" in some moleculsr/nanoscale fashion. Edison (talk) 03:10, 6 March 2011 (UTC)[reply]

energy

It is sometimes, but not always, possible to obtain "energy" via a differentiation of a spatial coordinate; (for example, kinetic energy is related velocity squared - which is a nonlinear operation on the first derivative of a spatial coordinate). So, "energy" is rarely a useful coordinate in most physical treatments. But you must know the mass to get the actual energy, right? — Preceding unsigned comment added by 98.221.254.154 (talkcontribs)

I would say that energy is very often a useful concept of most physical systems. Dauto (talk) 06:04, 5 March 2011 (UTC)[reply]
The OP is directly quoting my response to the question about "Dimensions" from March 2, 2011, but out of context. I was discussing whether "energy" would be a useful generalized coordinate, not whether it would be "useful." Nimur (talk) 13:32, 5 March 2011 (UTC) [reply]
Yes, you need to know the mass to get the (classical, kinetic) energy. 83.134.170.152 (talk) 06:32, 5 March 2011 (UTC)[reply]
Exactly - if you mean classical, kinetic energy, you must know the mass of the object - which is a constant. Nimur (talk) 13:35, 5 March 2011 (UTC)[reply]

Food spoilage rates

Why does meat spoil faster than fruits and vegetabless? And why do flour, cookies, potato chips, and crackers just go stale instead of spoiling like the aforementioned meat, fruits, and vegetables? —SeekingAnswers (reply) 08:03, 5 March 2011 (UTC)[reply]

A few thoughts:
1) Moisture content is critical, and dryer things may not rot at all, like crackers and flour.
2) Trans fats are specifically used because they increase shelf life, in products like cookies, potato chips, and crackers. Apparently they are as unhealthy for microbes as they are for us.
3) Hi salt content also retards bacterial growth.
4) In other cases, some partially rotten products may still be considered edible. This is the case with fruit, for example. There are examples of fermented meat which people still eat, but most people find such things more distasteful than fermented fruit.
5) In the case of fruit, vegetables, and, in particular, potatoes, then may remain "alive" for some period after they are removed from the rest of the plant, and thus their own immune system may continue to fight bacterial growth. Since potatoes and onions can grow new sprouts, under the right conditions, this means that they must still be alive.
6) Fruit often has a skin designed to act as a barrier to bacteria, and that provides some period of protection. StuRat (talk) 18:21, 5 March 2011 (UTC)[reply]
Trans fats are not intentionally added, dude. They are a byproduct of catalytic hydrogenation of alkenes. John Riemann Soong (talk) 22:59, 5 March 2011 (UTC)[reply]
I'm not following you. If they aren't intentionally added, are you saying they are accidentally added ? StuRat (talk) 07:07, 6 March 2011 (UTC)[reply]

Thanks, but the answer to my two questions are still not entirely clear. I'm rather confused by the listing out of so many factors; which of them are most important? I don't think the question of why meats are so much more prone to spoilage than fruits and vegetables has been completely addressed; salted meat will still spoil faster than fruits and vegetables. Anyone else have anything to add? —SeekingAnswers (reply) 01:27, 6 March 2011 (UTC)[reply]

Beef jerk is salted and dried meat. I lats a long time. Dauto (talk) 01:47, 6 March 2011 (UTC)[reply]

Run Lola Run “sideways thru time” question

In the film Run Lola Run, and in many other sci fi films, the theme of “how would a small difference in the order of events affect long-term consequences as time moves on?” is explored. In this film’s case, (and it takes its philosophy pretty seriously) , Lola has to run somewhere in order to save her boyfriend. That run is shown three times, and each time there is some small difference, for example, she bumps into some people which slows her down a little. That difference ends up completely changing the results of her efforts at the end of the day. No one seems to think there is anything odd about this, and other films explore this theme with similar outcomes.

I have thought about it, and in my thought experiment, the whole platform on which Run Lola Run rests can be demonstrated to be false. Yes, it is true that a single, small event can change a whole life. For example, a young man attends a party that he had not intended to go to, meets a girl, and they end up getting married. But such life changing events that are the result of a small deviation in routine are rare. Run, Lola, Run implies that EACH such deviation has long-term consequences that radically alter one’s life.

I feel that can be refuted decisively by reflecting on one’s own life. In my case, for example, I lived in the same place for over 10 years. Each day, something unexpected would occur; I’d meet an old friend by accident and go and have a few drinks and so on. But at the end of each day, however many deviations from the norm occurred, I would end up back in my own little bed, safe and sound. If small deviations in the norm changed outcomes like they do in Run Lola Run, one’s life would be a huge storm of chaos, each day would be a major alteration of direction. It would be impossible to live like that. I feel surprised that no one seems to mention this when the scientific and philosophical themes of such films are discussed: that the structure of lives has a sturdiness that is not NORMALLY perturbed by chance meetings and intervening events. We are put off course briefly but we get back on course. We miss a train, so we catch the next one. If that means we miss a meeting, we discuss it with someone else. So our lives end up pretty much where they were before.

I’ve got no argument with Run Lola Run in the depiction of a small PARTICULAR event having a radical impact on the outcome of an entire enterprise, but it is only feasible that it happens once. The film puts forward the idea that EVERY such deviation from the norm has a radical effect, but the general ordinariness of our own lives provides ample refutation of that notion. I think this is a scientific question as well as a philosophical one, and I would be interested in the opinions of other WIkipedians. Myles325a (talk) 09:01, 5 March 2011 (UTC)[reply]

Have you checked out butterfly effect and perhaps Butterfly effect in popular culture? Nil Einne (talk) 09:24, 5 March 2011 (UTC)[reply]
You might also wish to consider the sum over histories theory of Richard Feynman. --TammyMoet (talk) 11:37, 5 March 2011 (UTC)[reply]
For an even more detailed (and in my opinion more delightful) "study" of this question, see Groundhog Day, where the same day is run through hundreds of times with only one variable (Bill Murray) changing. In any case, the idea that the little things in life get overwhelmed (most of the time) by the big trends is indeed a valid criticism, I think. I'm not sure it holds universally, though. In the context of history, this is discussed in some detail in Carr's What Is History?, where he talks about attributing too much to "accidents" of history (e.g. the entire fate of the Russian revolution to the fact that Trotsky got a cold on the wrong day). Carr says that these explanations miss out on the fact that huge forces have been previously put into motion anyway behind these "accidents" — that focusing on the tip misses you the rest of the iceberg. --Mr.98 (talk) 14:15, 5 March 2011 (UTC)[reply]
I think the answer is somewhat in between. That is, while not all seemingly insignificant events in your life have a dramatic effect on the outcome, many do. I think of it in terms of equilibrium, and will use a glass bowl analogy. Which events upset the equilibrium, like pushing a bowl off a table where it shatters, and which are self-correcting, like pushing a bowl, having it wobble a bit, then settle back down ? Your "missing a train" example could be self-correcting, if you get on the next train. However, what you do during that period or what you would have done, had you been on the earlier train, are also events which need to get back onto the original path. The longer it takes, and the more subsequent events occur, before you "catch the next train", the less likely it is that the time-line will be fully restored. StuRat (talk) 18:10, 5 March 2011 (UTC)[reply]
The real problematic part here is strictly speaking, as far as we know, time just doesn't work this way. Film gives us the impression that we can divide it into little indefinite segments, and possibly reloop "time lines" in different ways. But as far as science tells us (odd interpretations of quantum theory notwithstanding), the arrow of time is one way (although it can run at different rates depending on your reference frame, etc.). This is not a pedantic point; it was the foundation of a great deal of the philosophy of time, by people such as Henri Bergson, who explicitly criticized film as giving people very bad ideas about how to visualize "time" and "events". Bergson would say about your comment: "there is no time-line to be restored; there is just one time-line, and it plays out how it plays out, irrespective of how you imagine it in a more ideal form." Only in fiction do we get to have multiple "time-lines" in the sense you mean. To very much bastardize Bergson, he would say, when you try to make broad statements about the nature of time based on metaphors from film, you are making as big a category error as you would if you were trying to make broad statements about the nature of violence based on cartoons. --Mr.98 (talk) 19:47, 5 March 2011 (UTC)[reply]
The OP seems to assume that there exists a sequence of events, called the norm, from which various degrees of deviation occur i.e. 1) deviations that cause only a temporary disturbance to the sequence that is supposed to be in equilibrium to which it returns, and 2) deviations that have expanding repurcussions that irrevocably change everything that follows. The assumption is appealing because it allows unlimited speculation about Counterfactual history. However it is impossible to prove the speculation that any future sequence of events is more "normal" than any other, and the grading of "deviations" is an Anthropomorphism. Example: the corrosion of an electric contact may cause an innocuous click in a telephone conversation or prevent a nuclear bomb exploding, but the corrosions were equally inevitable in the given environments. Cuddlyable3 (talk) 23:12, 5 March 2011 (UTC)[reply]
All physical systems respond elastically to perturbations below some threshold and plastically to larger perturbations. It's universal. I see no way to avoid the conclusion that human history, considered as a physical system, behaves in this way also. You could treat Earth's weather as external to the system, or you could treat it as internal and have the external perturbations come from micrometeorites or something. In the end it doesn't matter much.
You can't escape this by arguing that it's impossible to define a notion of alternate history, because that's an argument against the scientific method. If you believe that experimental data is noisy, and that one must ignore the noise when looking for patterns, then you believe in counterfactuals. That's what a counterfactual is. For that matter, if you believe that there's such a thing as entropy then you believe in counterfactuals. It's the same thing.
I realize that it's disturbing to think that human history could have gone entirely differently, and it's tempting to argue against it for that reason alone, but this is the science desk and that's the science. If you can come to terms with trillions of other galaxies that probably harbor intelligent life, or with the eventual demise (by heat death or spacetime singularity) of everything the human race has ever created, then you can come to terms with this. -- BenRG (talk) 01:55, 6 March 2011 (UTC)[reply]

Kolmogorov complexity

Hi. I'm pretty new to computer science, but am fascinated by the term Kolmogorov complexity. The article has an example that contains the letters "ab" repeated 32 times, and encodes it as "ab 32 times". However, what if the actual string given is "ab 32 times"? How could we differentiate that from an encoded version of the abab... string? 88.112.51.212 (talk) 12:13, 5 March 2011 (UTC)[reply]

The example we use in our article is a bit "invalid" (for the sake of simplicity) - it's missing a complete description of an escape character that would be required. Some piece of information (at least one single bit) is required to indicate whether the substring "32 times" should be interpreted literally or as a "modifier" to the preceding characters. This extra information can be included by using an escape-character that is always specially-interpretted; but then you can never print the escape character! So you need a little more sophistication in the scheme. This problem pops up a lot in the implementation of encoding-algorithms; the answer invariably is either "indicate the special interpretation using side-channel information" or "indicate the special interpretation by using in-band escape-characters." Naturally, trouble arises when trying to convey the literal value of the escape-character; but again, this can be done with either in-band or side-channel information.
You might find it helpful to consider actual implementations of run-length encoding or lossless data compression to see the practical implementations of these theoretical requirements.
I will also point out that a complete information-theoretical description of complexity of a data-stream should account for the complexity of both the data-stream and the algorithm or machine that must be used to decode it. Abstract machines (or decoding-algorithms) require some minimum number of bits to define them - and this is often the "forgotten" information-container when "infinite compression" schemes are proposed. You may find Turing completeness interesting, and the principle of the quine program concept. Nimur (talk) 13:45, 5 March 2011 (UTC)[reply]
It is not true that in constructing a data compression code one must invariably indicate the special interpretation by using in-band escape-characters. In LZW compression thst is used in GIF image files the coding algorithm starts by expanding all characters slightly rather than compressing them. The trick lies in the encoder then teaching (up to a limit) the decoder about every string of characters that might repeat, so that only one character suffices to convey each repetition. The resulting compression ratio can be very high. Cuddlyable3 (talk) 15:52, 5 March 2011 (UTC)[reply]
I think this talk about escape characters is a red herring. Escape characters are an irrelevant implementation detail, and they're not even commonly used in real compressed stream formats. The important thing is that compressed and uncompressed streams are kept separate. You don't have to distinguish your compressed stream from an identical uncompressed stream because your decompressor will never get an uncompressed stream. If you compress abababababababababababababababababababababababababababababababab to "ab" 32 times then you probably will compress "ab" 32 times to something like '"ab" 32 times' 1 time—or at any rate, to something other than a copy of the uncompressed string. Yes, the compressed string is longer than the uncompressed string; that's unavoidable in general (by the pigeonhole principle, aka the counting argument).
The other important thing is that the compressed formats used in discussions of algorithmic complexity have to be Turing-complete programming languages. Most (all?) real-world compressed formats aren't. The reason you need Turing completeness is that every compressed format must be "convertible" into any other by prepending a description of a decompressor for the other format to compressed data in that other format. This makes the optimum compression ratio format-independent in the limit of arbitrarily large files, which means that you can define algorithmic complexity (in the limit of arbitrarily large files) without tying it to any particular compressed format. -- BenRG (talk) 23:31, 5 March 2011 (UTC)[reply]

Are gays and lesbians fertile?

If a gay man had sex with a straight woman or a lesbian was raped by a straight man, would they produce viable children? —Preceding unsigned comment added by 93.29.254.150 (talk) 12:18, 5 March 2011 (UTC)[reply]

Most likely. Sexual orientation has no effect on fertility necessarily. See Clay Aiken (specific example) and artificial insemination (the method that he used). Finalius (Say what?) 13:09, 5 March 2011 (UTC)[reply]
Yes, of course they are. Many have had children. And the millions of those who were married but "in the closet" over the years had children as well. --Mr.98 (talk) 14:11, 5 March 2011 (UTC)[reply]
Residents of France, of whom the OP seems to be one, are not AFAIK famous for such ignorance about sexuality. The OP poses scenarios that are incompatible with consensual sex. A child conceived in such abnormal encounters is as likely to be biologically viable and grow as any other, but its mother will need to compensate somehow for the lack of a normal father figure. Cuddlyable3 (talk) 15:25, 5 March 2011 (UTC)[reply]
@Cuddlyable3: I don't know anything of mothers having to compensate for the lack of a normal(?) father. Actually, I don't even know what a normal father is.Quest09 (talk) 02:22, 6 March 2011 (UTC)[reply]
Also, the first case can perfectly well be consensual. Happens now and then. Aaronite (talk) 03:11, 6 March 2011 (UTC)[reply]
Although gay parents are not unknown, I don't know if they are equally fertile. If their hormonal levels are different, they certainly could be less fertile. Quest09 (talk) 02:23, 6 March 2011 (UTC)[reply]
@Quest09, on this ref. desk we try to provide sourced information and references to questioners. You have posted "I don't know..." 3 times[8] [9] which is unhelpful. Cuddlyable3 (talk) 04:36, 6 March 2011 (UTC)[reply]
@Cuddlyable3: Saying "i don't know" is a polite way of saying "I've never heard about this shit before."Quest09 (talk) 13:18, 6 March 2011 (UTC)[reply]

Skyscraper

Let's say, in strictly theoretical terms, that there is a skyscraper so tall it reaches all the way up into the higher atmosphere, space even. Would the building look like it was bending upwards from ground level according to the Coriolis effect, or would it still look perfectly straight? Thank you.--Editor510 drop us a line, mate 14:33, 5 March 2011 (UTC)[reply]

A viewer standing on the ground is in the same inertial frame of reference as the skyscraper and therefore observes its true straight shape, with minor modifications due to obscuration by clouds, and refraction due to the atmosphere's temperature gradiant. Cuddlyable3 (talk) 15:31, 5 March 2011 (UTC)[reply]
A viewer standing on the ground is in the same inertial frame of reference as the base of the skyscraper. Except at the poles, a skyscraper on the surface of the rotating Earth will be experiencing a slightly different acceleration at each height. TenOfAllTrades(talk) 16:02, 5 March 2011 (UTC)[reply]
So? Cuddlyable3 (talk) 21:51, 5 March 2011 (UTC)[reply]
As Coriolis effect says, the Coriolis effect is an apparent deflection of moving objects when they are viewed from a rotating reference frame. The skyscraper is not moving in the reference frame of somebody standing besides it, so there is no Coriolis effect. However, if somebody dropped an object from the top then, due to the Coriolis effect, it wouldn't fall straight down. This can be observed from far smaller heights. A gunner or sniper at the bottom aiming for the top would also see the projectile curving and would have to not aim straight at the goal even if gravity is ignored. PrimeHunter (talk) 17:47, 5 March 2011 (UTC)[reply]
Thank you.
Resolved
--Editor510 drop us a line, mate 22:29, 5 March 2011 (UTC)[reply]

Will the frame dragging effect of twisted spacetime cause a theoretical if not appreciable bend in a similarly tall object? —Preceding unsigned comment added by 129.67.37.227 (talk) 15:01, 6 March 2011 (UTC)[reply]

Looking for an electronic engineering journal

Hi there, hope this is the right place to ask! I was wondering if someone could recommend a good journal on electronic engineering, preferably one that can be subscribed to in paper form from the UK. Kind regards, Raywil (talk) 19:54, 5 March 2011 (UTC)[reply]

Are any of these any good to you? List of Institute of Electrical and Electronics Engineers publications--Aspro (talk) 20:19, 5 March 2011 (UTC)[reply]
Thanks for replying. There are definitely some interesting titles there and I didn't realise the IEEE published so many things. I'm still looking for a particular recommendation though if anyone has one. Raywil (talk) 21:11, 5 March 2011 (UTC)[reply]
Electronic Design Magazine is free and though Electronics (magazine) 1930 - 1995 is no longer published you may find copies in library collections. Cuddlyable3 (talk) 21:59, 5 March 2011 (UTC)[reply]
Any recommendation would really depend on what the OP is hoping to gain from it. If it is just for general interest, then pop along to a local WHS and paw through a copy of Electronics World. If it is to prepare for an electronics course then it would be best to ask the course lecturer or whoever for their recommendation. Local public reference libraries should have a subscription for one or two of the more popular publications. Or let your imagination rip! One of my student flat-mates, got his dear aunt to fork out for a Playboy subscription. After all - he was studying anatomy (an' 'e ended up with first class 'onours to boot!).--Aspro (talk) 22:57, 5 March 2011 (UTC)[reply]
Thank you very much for all of your suggestions so far. I've sent off for a copy of Electronic Design and will have to look out for Electronics World next time I'm in WHS as I've never noticed it there before. I'm considering studying the subject at uni so thought I'd better get myself some related reading material. Any more appropriate suggestions would definitely be much appreciated. Raywil (talk) 00:54, 6 March 2011 (UTC)[reply]
IEEE Spectrum is a good one too - it's not too technical or domain-specific, but provides a "generalist" viewpoint into the cutting edge of electronics. I would say Spectrum is the magazine you should read (IEEE calls it the "flagship publication"). You can read online versions at http://spectrum.ieee.org or order paper subscriptions. Nimur (talk) 15:33, 6 March 2011 (UTC)[reply]

adaptive features of fish

tell me about adaptive features of fish —Preceding unsigned comment added by 220.225.96.217 (talk) 20:35, 5 March 2011 (UTC)[reply]

Have you tried searching the web or searching for relevant articles on Wikipedia? We are somewhat unlikely to write an essay on the subject for you. Please ask us about specific issues when you have done your own research. 88.112.59.31 (talk) 21:03, 5 March 2011 (UTC)[reply]

inverting integrators and time integral of voltage

When I take the time integral of voltage, what unit do I get? I'm trying to use an equation similar to that in Operational_amplifier_applications#Inverting_integrator and I want to know if I have an RC value of 1.06 milliseconds how much that would scale my output by. John Riemann Soong (talk) 21:54, 5 March 2011 (UTC)[reply]

You can get the Time constant you mention using R = 1060 ohm and C = 1 uF. If there is a step change of +v at the input then during the next 1.016ms (assuming the integrator output is not saturated) the output will change negatively by −0.632v. Cuddlyable3 (talk) 22:16, 5 March 2011 (UTC)[reply]
Can you walk me through that? I totally understand the RC part -- what I don't get is that is that if I take this integral of a sinusoidal function, I seem to get an output integral reported in V/s, not V. This is because the inverted integral of sin t dt is cos t -- thus I get cos(t2)/RC-cos(t1)/RC as my theoretical output.
Which of course I am puzzled by, because my output should be in V, not V/s. If I do a square wave I am fine -- the inverted integral of a constant +v is -v*t/RC, which gives me an output in volts. How do I calculate how to scale my output for sinusoidal functions? John Riemann Soong (talk) 22:36, 5 March 2011 (UTC)[reply]
The sinewave input is Vin = Vpk sin ( ω t ) where ω = 2 π f
You omitted the frequency f. The angular velocity ω is in radians sec-1. Cuddlyable3 (talk) 00:09, 6 March 2011 (UTC)[reply]
Exactly. In fact sin t is not a physically meaningful statement because it is a non-polynomial function of a dimensional quantity which makes no sense. Dauto (talk) 02:04, 6 March 2011 (UTC)[reply]
This non-polynomial?
Cuddlyable3 (talk) 05:06, 6 March 2011 (UTC)[reply]
Yes, it is non-polynomial. see the first sentence of polynomial. The problem is that every term in the expansion above has a different dimension (it is measured with different units) so they cannot be added in a physically meaningful way. Dauto (talk) 06:17, 6 March 2011 (UTC)[reply]

Image of a moleclue

OK, how would the researchers this image? After all I've learned of quantum mechanics, I thought it wasn't possible to narrow down the location of an atom so closely. Magog the Ogre (talk) 23:02, 5 March 2011 (UTC)[reply]

That's an electron density map. You can theoretically predict it using calculators. It's important because we can now measure electron density empirically rather than merely theoretically. John Riemann Soong (talk) 23:11, 5 March 2011 (UTC)[reply]
As long ago as 1989, IBM used a scanning tunneling microscope to position 35 Xenon atoms to spell out IBM. SpinningSpark 01:04, 6 March 2011 (UTC)[reply]

Then what is all this quantum physics nonsense about not knowing an atom's state at any place and time? Magog the Ogre (talk) 02:51, 6 March 2011 (UTC)[reply]

See quantum uncertainty. Bear in mind that the scale of an atom is very different from the scale of a nucleon or an electron, and that while quantum uncertainty is quite real at all physical scales, it's only really relevant once you're subatomic. — Lomn 04:04, 6 March 2011 (UTC)[reply]

So that means a telescope that's 10000x as powerful as this one could probably not exist? Magog the Ogre (talk) 08:21, 6 March 2011 (UTC)[reply]

Microscope, not telescope. Unlikely, for different reasons. Plasmic Physics (talk) 08:36, 6 March 2011 (UTC)[reply]
It's also worth noting that you can't really tell the precision of those images very well from the image alone. At the level of an atom, uncertainty principle says there will be limits to your precision — not that you can't know anything. It's a ratio of how precise your understanding of both position and momentum can be. It is only when you get to the level of, say, a bare electron, that the ratio gets so extreme that you start being able to know only one or the other. It is a common entry-level physics problem to ask students to calculate how much precision in position/momentum you can have for visualizing things on that level, how "in focus" they can maximally be. --Mr.98 (talk) 14:36, 6 March 2011 (UTC)[reply]

March 6

Better quality rest giving better quality work

Your demanding day job makes you feel tired. Is there any scientific evidence (rather than speculation please) that better rest in the evening results in better work the following day? If you spend evenings feeling bored and physically uncomfortable, is there any significant difference work-wise from evenings when you are entertained and physically comfortable? Thanks 92.29.117.180 (talk) 00:27, 6 March 2011 (UTC)[reply]

The articles Sleep deprivation and Effects of sleep deprivation on cognitive performance provide reliable sources. Cuddlyable3 (talk) 01:32, 6 March 2011 (UTC)[reply]
I think the OP is talking about how well you relax in leisure time, not how well you sleep. 90.195.179.167 (talk) 02:25, 6 March 2011 (UTC)[reply]
It is tritely obvious that the OP is not interested in how I personally sleep. Sleep is the major remedy for feeling tired and its quantitative relation to work efficiency has been studied extensively, whereas subjective evening notions of boredom, entertainment and discomfort do not provide hard scientific evidence. Cuddlyable3 (talk) 04:58, 6 March 2011 (UTC)[reply]

I was not asking about sleep, thanks. Let's assume you are getting enough sleep. 92.15.18.16 (talk) 12:16, 6 March 2011 (UTC)[reply]

Where is water taken up in the digestive system?

Where is uptake of water in the human digestive system mainly taking place? The stomach, the small intestine or somewhere else?

From digestion: "Other small molecules such as alcohol are absorbed in the stomach, passing through the membrane of the stomach and entering the circulatory system directly." The water molecule is even smaller than alcohol, so I presume water is taken up by the stomach.

From small intestine: "Water and lipids are absorbed by passive diffusion throughout the small intestine."

--Mortense (talk) 00:58, 6 March 2011 (UTC)[reply]

The stomach and the small intestine will absorb some water but the large intestine is where the bulk of the water is absorbed. Dauto (talk) 02:16, 6 March 2011 (UTC)[reply]
Agreed. Otherwise the intestinal contents would be too thick to flow around the sharp bends of the small intestine. StuRat (talk) 06:55, 6 March 2011 (UTC)[reply]
See colon, the first line confirms Dauto's answer. Richard Avery (talk) 08:51, 6 March 2011 (UTC)[reply]

Two questions about filters

(1) Why do filters for digital cameras exist? Can't you process the pictures after shooting them with a software filter?

(2) Is there a filter for glasses to make everything look black and white? Quest09 (talk) 02:26, 6 March 2011 (UTC)[reply]

1)You can process color images with Photoshop or other software and gain many of the advantages of using color filters, such as if the image is transformed to black and white, after a yellow filter effect is used to emphasize clouds in an outdoor color photo. Actual filters can achieve some effects which would be difficult or impossible with Photoshop, since a filter might have a very narrow bandwidth, while the three channels of color info in a digital image are very broadband. 2)Kodak produced a "viewing filter" which made it easier to visualize how a B & W image of a color scene would look. The name of it is a "Kodak Wratten 90 filter." If memory serves, it might have been brown or orange. The scene did not really look black and white through it; that is impossible. See also [10]. Edison (talk) 03:07, 6 March 2011 (UTC)[reply]
UV, infra-red and polarizer filters cannot be emulated in software. Cuddlyable3 (talk) 05:17, 6 March 2011 (UTC)[reply]
1) One other advantage would be to get the filtered results immediately, and thus know if another photo should be taken, while the subjects are present and the lighting is still the same. Here I'm assuming that the software filter is used later on. If you apply it as the picture is taken, then that's good, too. StuRat (talk) 06:43, 6 March 2011 (UTC)[reply]

If there's a heart defibrillator, why isn't there a brain defibrillator?

So a defib paddle can restart a flatlined heart; why isn't there a defib device that restarts a flatlined brain?

What would it take to invent a device that would restart a flatlined brain? What obstacles would it have to overcome and how would it overcome them? --70.179.169.115 (talk) 03:39, 6 March 2011 (UTC)[reply]

The heart has a beat to restart, the brain doesn't. Clarityfiend (talk) 04:31, 6 March 2011 (UTC)[reply]
CF, what does the brain have to restart then? It must have something to keep going. --70.179.169.115 (talk) 04:34, 6 March 2011 (UTC)[reply]
DISCLAIMER: I'm no expert, so take the following with a boulder of salt. Compared to the brain, the heart and its operation is a lot less complicated. It follows a repetitive, relatively simple cycle, which somebody has discovered that a jolt of electricity can restart. Various areas of the brain, on the other hand, fire off at different times, intensities and combinations depending of the stimulus. There is no set pattern AFAIK for you to kick start. Also, I'm guessing that neurons are a lot more fragile than heart muscle, and would not take kindly to being zapped. As a rough analogy, kicking a malfunctioning washing machine has a much better chance of working than (physically) booting your PC. Clarityfiend (talk) 05:16, 6 March 2011 (UTC)[reply]
Actually, this question isn't entirely off base. The heart doesn't usually stop beating entirely, but rather goes into an abnormal contraction pattern, called fibrillation, and the shock stops that and allows the normal beat to return. Similarly, the brain sometimes has abnormal electrical patterns, and some attempt has been made to stop those and allow the brain to "restart", via electroshock therapy. While this therapy has a bad reputation from being used improperly, it still has it's advocates, especially when compared with brain surgery. StuRat (talk) 06:37, 6 March 2011 (UTC)[reply]

It's an interesting question. I know little about it other than you would need to get the the neuronal ion pumps to start working again but the IP may be interested in having a read around the subject of cortical spreading depression in the visual cortex. It's kind of like part of the brain flatlining for a while (in the sense that neurons remain excessively inhibited) after a hyperexcitability phase (a spreading jolt of electricity). Sean.hoyland - talk 07:09, 6 March 2011 (UTC)[reply]

What does it mean to re-start a brain? We re-start heart because they are fibrillating and you could die. If the brain is not working properly there are other means to 're-start' it. If you mean re-starting a brain without activity, I have to say that you should have re-started the heart earlier. —Preceding unsigned comment added by 212.169.187.20 (talk) 13:08, 6 March 2011 (UTC)[reply]

weather

If we wanted to know everything about the weather to predict it w/ 100% accuracy, would it be better to ask all the inhabitants to hold their breath and stop moving or include those interactions in our analysis? by better i mean to predict the weather, not for the inhabitants. —Preceding unsigned comment added by 98.221.254.154 (talk) 04:42, 6 March 2011 (UTC)[reply]

What about the animals, continental plate movement, variable solar activity, the odd earthquake, even plant growth? Clarityfiend (talk) 05:21, 6 March 2011 (UTC)[reply]
How would you even model interactions between humans and the atmosphere? Titoxd(?!? - cool stuff) 07:39, 6 March 2011 (UTC)[reply]
You could build a scalar field of global baked bean sales. That would be a start. Sean.hoyland - talk 07:56, 6 March 2011 (UTC)[reply]
To actually answer the question, no, because of the butterfly effect.--Shantavira|feed me 08:18, 6 March 2011 (UTC)[reply]

Testosterone as an anti-depressant in men

Has any research been done on using testosterone as an anti-depressant on men with or without low testosterone levels? —Preceding unsigned comment added by 76.169.33.234 (talk) 05:55, 6 March 2011 (UTC)[reply]

There's plenty of research showing a small but statistically significant decrease in bioavailable testosterone levels correlated with depression (such as [11]), but the only research I can find on treatment of depression with testosterone has actually been done on women to treat symptoms of menopause (such as [12], if you're allowed to access it). Someguy1221 (talk) 06:05, 6 March 2011 (UTC)[reply]

Period & Bloating

When a woman gets bloated during her period, is the bloating all blood or water? —Preceding unsigned comment added by 76.169.33.234 (talk) 05:57, 6 March 2011 (UTC)[reply]

It's water retention. Surprisingly little blood is lost during the menstrual period - about a teacup over the average 5 days. Most of what you see is endometrium, or the lining of the womb. --TammyMoet (talk) 09:13, 6 March 2011 (UTC) Edit: Sorry, I meant an eggcup. The Menstrual cycle article says 10 - 80 ml. (Note to self: wake up before you start posting!) --TammyMoet (talk) 11:32, 6 March 2011 (UTC)[reply]